Gram-Negative Bacteria MCQ

Lakukan tugas rumah & ujian kamu dengan baik sekarang menggunakan Quizwiz!

*The answer is A.* The major determinant of virulence in H. influenzae is the presence of a capsule. There is no demonstrable exotoxin, and the role of endotoxin, and the role of endotoxin is unclear. while one would expect that IgA protease would inhibit local immunity, the role of this enzyme in pathogenesis is as yet unclear. Flagella is not considered a virulence factor.

A 2-year-old boy who missed several scheduled immunizations presents to the emergency room with a high fever, irritability, and a stiff neck. Fluid from a spinal tap reveals 20,000 white blood cells per milliliter with 85% polymorphonuclear cells. Gram stain evaluation of the fluid reveals small pleomorphic gram-negative rods that grow on chocolate agar. If an inhibitor is designed to block its major virulence, which of the following would be the most likely major virulence factor? a. Capsule formation b. Endotoxin assembly c. Exotoxin liberator d. Flagella synthesis e. IgA protease synthesis

*The answer is C.* Borrelia burgdorferi, the causative agent of Lyme disease, has a unique genome composed of a linear chromosome and a complement of circular and linear plasmids. The outer sheath of the spirochetes is also relatively unique in that it does not contain lipopolysaccharide, or endotoxin. The outer surface does not contain mycolic acids, which are found in the Mycobacterium species. B. burgdorferi does not produce any known exotoxins and is transmitted to humans by the bite of a tick. Many patients, however, do not know that they have been bitten by a tick until the characteristic rash appears. The highest incidence of Lyme disease caused by B. burgdorferi is in the Northeast and upper Midwestern United States.

A 13-year-old boy, previously healthy, developed flulike symptoms including fever and malaise. These constitutional symptoms were accompanied by a spreading, circular rash on the child's back. Travel and recreational history indicated that the boy had recently been camping in rural Connecticut. The boy was unaware of any abrasions, bites, or other injury. Which of the following characteristics is unique to the organism that is the most likely cause of this infection? A. The outer membrane contains lipopolysaccharide. B. The outer surface is composed of mycolic acids. C. The genome is composed of one linear chromosome and a series of circular and linear plasmids. D. The disease is caused by elaboration of a potent exotoxin. E. The disease is transmitted by the bite of a body louse.

*The answer is B.* This patient has Neisseria meningitidis, which coudl have been prevented with a Meningococcal conjugate vaccine, 4-valent.

A 21-year-old woman who is a college student is brought to the emergency department 2 hours after the onset of fever, chills, severe headache, and confusion. Her temperature is 39°C (102.2°F), respirations are 16/min, and blood pressure is 100/60 mm Hg. Physical examination shows numerous petechial lesions over the upper and lower extremities. There is resistance to neck flexion. Analysis of cerebrospinal fluid shows numerous leukocytes and gram-negative diplococci. Administration of which of the following vaccines is most likely to have prevented this patient's condition? (A) Haemophilus influenzae type b vaccine (B) Meningococcal conjugate vaccine, 4-valent (C) Pneumococcal conjugate vaccine, 7-valent (D) Pneumococcal polysaccharide vaccine, 23-valent (E) Varicella vaccine

*The answer is E.* The case described is typical for typhoid fever caused by Salmonella typhi. This multiorgan system disease is due to the dissemination of the microbe within macrophages. S. typhi enters the intestine, infects local macrophages, and is transported around the body, replicating in macrophages in the lymph nodes, spleen, liver, and bone marrow. The infection is accompanied by high fever and evidence of toxemia. Replication of the organism in Peyer patches leads to inflammation of the bowel wall and subsequent constipation. Bacterial emboli to the skin cause the rose spots seen on the patient's chest and neck. S. typhi is isolated on specialized media such as Hektoen agar on which the production of hydrogen sulfide by Salmonella is demonstrated as a black center within the colony. None of the other bacterial diseases cause the symptoms or have the microbiological characteristics described in the case. Cholera, caused by the Gram-negative, curved rod Vibrio cholerae, is characterized by watery diarrhea. Hemolytic uremic syndrome, caused most commonly by E. coli O157:H7, is characterized by bloody diarrhea, kidney failure, and thrombocytopenia. Shigellosis is generally associated with bloody diarrhea. While Shigella and Salmonella are both grown on Hektoen agar, Shigella produces greenishblue colonies without black centers. Tularemia can manifest in different ways, including ulcers with regional lymphadenopathy, eye infections, pneumonia, and a typhoidal form characterized by fever, malaise, myalgias, and weight loss.

A 20-year-old male is brought to the emergency department with a 1-day history of delirium. He has had a sustained fever of up to 40 °C and a history of progressive headache, myalgia, and constipation which began 10 days previously as he was returning to the United States from a trip to visit relatives in India. Physical examination revealed hepatosplenomegaly, diffuse abdominal tenderness, and red spots on the chest and neck. Colonies of a Gram-negative bacillus that produced a characteristic "fish-eye" growth (lactose nonfermenter with sulfur reduction) are isolated on Hektoen agar from blood and stool samples as shown in the photograph. What is the most likely diagnosis? (A) Cholera (B) Hemolytic uremic syndrome (C) Shigellosis (D) Tularemia (E) Typhoid fever

*The answer is A.* Since all choices are in italics, all are genus names and you do not need to look for a family name. Since all Enterobacteriaceae generally produce nitrate reductase, they all eventually produce a positive dipstick nitrite test. Escherichia is the only choice belonging to this family. Other members of Enterobacteriaceae that cause urinary tract infections include Proteus and Klebsiella. Staph and Strep are Gram-positive and Vibrio is a genus of Gram-negative, comma-shaped bacteria. A positive nitrite test rules out Staph. saprophyticus as the cause of a urinary tract infection.

A 24-year-old female presents with dysuria, as well as urinary urgency and frequency. A urine dipstick test is positive for both leukocyte esterase and nitrites. What genus or family is noted for the production of nitrites? (A) Escherichia (B) Staphylococcus (C) Streptococcus (D) Vibrio

*The answer is C.* Helicobacter pylori is a urease-positive organism that causes duodenal ulcers.

A 35-year-old man comes to the physician for a follow-up examination. He has had persistent left upper quadrant abdominal pain for 3 weeks despite therapy with omeprazole. Upper esophagogastroduodenoscopy shows an active duodenal ulcer. Tests of gastric and duodenal biopsy specimens for urease are positive. Which of the following is the most likely causal organism? (A) Campylobacter jejuni (B) Clostridium difficile (C) Helicobacter pylori (D) Proteus vulgaris (E) Salmonella typhi

*The answer is E.* Epiglottitis is a medical emergency requiring hospitalization. It can be fatal in 24 hours. Pediatric cases were almost always caused by H. influenzae type b and have been dramatically reduced by the conjugate vaccine.

A 3-year-old girl presents with difficulty breathing and will not lie down to be examined. You suspect acute bacterial epiglottitis and examine the child's epiglottis, which is highly inflamed. Which vaccine are you most likely to find that the child is missing? (A) Diphtheria (B) Neisseria meningitidis (C) Polio (D) Streptococcus pneumoniae (conjugate vaccine) (E) Haemophilus influenzae

*The answer is D.* The two most common bacterial causes of foodborne illness in the United States are Campylobacter and Salmonella. Both are associated with undercooked poultry and cause invasive (inflammatory) GI disease with the induction of fecal leukocytes. Salmonella are Gram-negative rods that are unable to ferment lactose. Campylobacters are motile, curved, Gram-negative rods that are isolated under microaerophilic conditions (5% oxygen, 5% to 10% carbon dioxide) on selective media such as Skirrow agar.

A 37-year-old man presented with a 2 day history of low grade fever, abdominal cramps diarrhea, nausea, and vomiting. One day prior to the onset of symptoms, six family members had eaten at an all- you-can-eat fried chicken buffet. Two other members of his family (his 82-year-old mother and a 14-month-old son) had similar symptoms. Examination of a fecal smear from the patient revealed abundant fecal leukocytes. Which of the following laboratory profiles is consistent with the most likely microbial cause of this man's condition? (A) Aerobic, β-hemolytic, spore-forming Grampositive rod (B) Bacitracin-sensitive, Gram-negative cocci (C) Coagulase-negative, Gram-positive cocci (D) Nonlactose fermenting, Gram-negative bacilli (E) Oxidase-positive, motile, curved, Gram-positive bacilli

*The answer is B.* Decreased gastric acid increases susceptibility to GI disease caused by Salmonella and Vibrio. The elderly and young children are also at an increased risk of infection.

A 37-year-old man presented with a 2 day history of low grade fever, abdominal cramps diarrhea, nausea, and vomiting. One day prior to the onset of symptoms, six family members had eaten at an all- you-can-eat fried chicken buffet. Two other members of his family (his 82-year-old mother and a 14-month-old son) had similar symptoms. Examination of a fecal smear from the patient revealed abundant fecal leukocytes. Which of the following medications, if taken by the man in the above case, could have increased his susceptibility to the infection he developed? (A) Acetaminophen (B) Antacids (C) β-Blockers (D) Ibuprofen (E) Statins

*The answer is B.* Patients with indwelling bladder catheters are at increased risk for urinary tract infections caused by both typical and opportunistic organisms. Typical organisms include Escherichia coil, Klebsiella pneumoniae, Staphylococcus saprophyticus, and Proteus mirabilis. This question asks you to identify an opportunistic organism according to microbiology. We can identify Pseudomonas aeruginosa as the pathogen because it is the only non-lactose fermenting gram-negative rod listed. It is also a common cause of urinary tract infections in patients with indwelling bladder catheters. (Choice A) E. coil is a lactose-fermenting gram negative rod and the leading cause of ur nary tract infections (UTI). (Choice C) K. pneumoniae is also a lactose-fermenting gram-negative rod. In addition to UTI it is a leading cause of spontaneous bacterial peritonitis and nosocomial pneumonias. (Choice D) Staphylococcus saprophyticus is a gram-positive coccus that typically causes UTI in sexually active women. It has a seasonal distribution, being worse in the summer. (Choice E) An enterococcus cloaca is a gram-positive coccus in the Streptococcus family. It is also a common cause of opportunistic UTI. Vancomycin-resistant Enterococcus is a particularly fearsome opportunistic organism. (Choice F) U. urealyticum is a known cause of urethritis but it requires special media rich in urea and cholesterol to grow.

A 39 year-old paraplegic man with an indwelling bladder catheter presents to the emergency room complaining of twenty-four hours of rigors nausea, and vomiting. His temperature is 38.9°C (102.0°F). Urinalysis shows 3+ leukocyte esterase and numerous white blood cells. Urine and blood cultures grow non-lactose fermenting gram-negative bacilli. Which of the following pathogens is the most likely culprit? A. Escherichia coil B. Pseudomonas aeruginosa C. Klebsiella pneumoniae D. Staphylococcus saprophyticus E. Enterococcus cloacae F. Ureaplasma urealyticum

*The answer is D.* This organism is the cause of RMSF. B. microti is transmitted by ticks and causes babesiosis; O. tsutsugamushi is transmitted by mites and causes scrub typhus. R. prowazeki is transmitted by lice and causes epidemic typhus. R. typhi is transmitted by fleas and causes endemic typhus.

A 40-year-old man presented with a rubeola-like rash on the extremities, chills, fever, myalgia, and malaise 5 days after returning from a June fishing trip in Arkansas. A history of tick bites is noted and Rocky Mountain spotted fever (RMSF) is suspected. What is the etiologic agent of this disease? (A) Babesia microti (B) Orientia tsutsugamushi (C) Rickettsia prowazeki (D) Rickettsia rickettsii (E) Rickettsia typhi

*The answer is D.* Sal. typhi has only human hosts.

A 45-year-old man who recently returned from Africa has been febrile for several days and now presents with abdominal pain. His blood cultures grow out Salmonella typhi. What was the most likely source of his infection? (A) Raw chicken (B) Undercooked hamburger (C) Contact with baby goats on a farm and then eating without washing hands (D) A food preparer with bad personal hygiene (E) Undercooked pork

*The answer is C.* Typhoid fever is common in most parts of the world except in industrialized regions such as the United States, Canada, Western Europe, Australia, and Japan. In developed nations typhoid fever is generally not suspected unless there is a history of recent travel to areas where the disease is endemic. Over the past several years, travelers to Asia, Africa, and Latin America have been especially at risk. Typhoid fever (also referred to as "enteric fever") is a life-threatening illness caused by the bacterium Salmonella typhi or Salmonella paratyphi. Other species of Salmonella are not associated with typhoid fever. Typhoid fever is a fecal-oral disease that begins after ingestion of S. typhi or paratyphi. These organisms penetrate the gut mucosa both via transporters on enterocytes and via phagocytosis by M cells in Peyer's patches. The organisms are then phagocytosed by macrophages, within which Salmonella (para) typhi are specially adapted to survive. Macrophages carry the infective organisms to the liver, spleen, and bone marrow. Hepatosplenomegaly from organism growth ensues. From here, these species are able to cause bacteremia and sepsis. Salmonella typhi and paratyphi also colonize the gallbladder, which allows access to the gut lumen on a virtually limitless basis. Within the gut lumen, S. typhi and paratyphi do more than disseminate—they can cause drastic inflammation within Peyer's patches, causing intestinal hemorrhage as well as potential gut perforation which can cause polymicrobial peritonitis and sepsis the mechanisms by which typhoid fever can cause death. Patients who do not experience fulminant disease are at risk for becoming chronic carriers of the bacterium (such as Typhoid Mary) and can unknowingly affect dozens of other people. S. typhi and paratyphi are shed from the bile into the stool. Contaminated water and the handling of food with unwashed hands then allow the bacteria to be ingested. There is no animal reservoir for this disease. The common clinical picture of this disease is mild abdominal cramping with a low fever and diarrhea OR constipation initially. Subsequently the patient can develop salmon-colored "rose spots" on the abdomen develop hepatosplenomegaly and recolonization of the gut leading to hemorrhagic diarrhea and sepsis. None of the other answer choices cause the clinical scenario mentioned above.

A 46-year-old male who just returned from a mission trip to Latin America presents to your office complaining of fever myalgias, dull abdominal pain, and a week-long history of watery diarrhea that has recently become bloody. Physical examination reveals a fever of 38.9°C (102.0°F), hepatosplenomegaly, and rose-colored spots at the periumbilical area. Which of the following is the most likely cause of this patient's symptoms? A. Escherichia coil B. Klebsiella pneumoniae C. Salmonella typhi D. Salmonella arteritides E. Campylobacter fetus F. Shigella flexneri G. Proteus mirabilis H. Yersinia enterocolitica I. Haemophilus influenzae J. Helicobacter pylori K. Vibrio cholerae L. Streptococcus pyogenes M. Streptococcus pneumoniae

*The answer is C.* Hemolytic uremic syndrome (HUS) is characterized by microangiopathic hemolytic anemia thrombocytopenia and renal insufficiency. It can occur after gastrointestinal infection caused by the Escherichia coil strain 0157:H7, a special strain of enterohemorrhagic E. coil (EHEC). The usual clinical picture associated with EHEC is that of a hemorrhagic colitis with hemorrhagic diarrhea and severe abdominal cramping caused by EHEC's ability to secrete a toxin similar to that of Shiga toxin. HUS tends to occur most commonly in children under 10 years old and in association with treatment of EHEC gastroenteritis with antibiotics. Most cases of HUS associated with EHEC 0157:H7 have been associated with eating undercooked, contaminated ground beef. Person-to-person contact in families and childcare centers is also an important mode of transmission. Infection can also occur after drinking raw unpasteurized milk and after swimming in or drinking sewage-contaminated water. Thoroughly cooking ground beef avoiding unpasteurized milk, and washing hands carefully during food preparation can prevent infection.

A 5-year-old Caucasian male is brought to the ER with somnolence, lethargy and oliguria. He developed diarrhea several days ago that later became frankly bloody. Laboratory studies show elevated blood urea nitrogen and creatinine. Peripheral blood smear reveals fragmented erythrocytes. This patient's condition is most likely related to consumption of which of the following foods? A. Custard B. Mayonnaise C. Undercooked beef D. Fried rice E. Oysters F. Eggs G. Canned beans

*The answer is B.* A major survival and virulence factor of Helicobacter pylori is urease, which neutralizes stomach acid to allow the organism to survive to reach the tissue.

A 62-year-old woman presents with signs of a gastric ulcer. She does not regularly take non-steroidal anti-inflammatory agents. Which characteristic appears to play a central role in the organism's ability to survive transit of the lumen to colonize the stomach? (A) Phospholipase-C production (B) Urease production (C) Microaerophilic lifestyle (D) O antigens

*The answer is B.* Patients infected with H. pylori are at risk of gastric adenocarcinoma.

A 52-year-old woman presented with indigestion and heartburn occurring shortly after meals which she treated with over-the-counter antacids. Physical examination revealed mild epigastric tenderness. A radiolabeled-urea breath test is positive. What cancer is the patient in the above case at risk for as a consequence of this infection? (A) Pancreatic cancer (B) Gastric adenocarcinoma (C) Hepatoma (D) Colon cancer (E) Esophageal cancer

*The answer is F.* The question stem describes an immunosuppressed (neutropenic) male who is experiencing symptoms consistent with bacterial sepsis. In cases of impaired humoral immunity, there is increased susceptibility to infections with P. aeruginosa as well as other pathogens. The lesions illustrated in the slide are consistent with ecthyma gangrenosum, a skin finding that is strongly associated with bacteremia by P. aeruginosa. These lesions result from perivascular bacterial invasion of arteries and veins in the dermis and subcutaneous tissue with subsequent release of exotoxins that are destructive to human tissue. Enzymes produced by P. aeruginosa such as Exotoxin A (protein synthesis inhibition) Elastase (degrades elastin — important for blood vessel destruction) Phospholipase C (degrades cellular membranes) and pyocyanin (generates reactive oxygen species) are recognized as important virulence factors and play a role in causing the vascular destruction and cutaneous necrosis known as ecthyma gangrenosum.

A 55-year-old male is being treated with combination chemotherapy for acute myeloid leukemia. His last complete blood count shows a white blood cell count of 800/cmm and a platelet count of 89000!cmm. He is now admitted with fever, chills and shortness of breath. His pulse is 120/min and BP is 90/40 mm Hg. Physical examination shows the following skin findings. Based on the clinical picture, which of the following bacteria is most likely responsible for this patient's symptoms? A. Escherichia coli B. Kelsiella pneumoniae C. Streptococcus pyogenes D. Streptococcus pneumoniae E. Haemophilus influenzae F. Pseudomonas aeruginosa

*The answer is D.* Urease breaks down urea, releasing ammonia that, when converted to ammonium, raises the urine pH. This leads to the formation of struvite stones. Although the etiological organism (see next question) can be involved in epididymitis, prostatitis, immune complex deposition, and blood stream infections, it is more likely to cause the formation of struvite stones.

A 78-year-old hospitalized male with an indwelling catheter develops a new onset of fever. The urine sample has an alkaline pH. The bacterium isolated from the urine is a urease-producing, highly motile, Gram-negative bacillus. What is the most likely complication of this infection? (A) Deposition of antigen-antibody complexes in the glomeruli (B) Development of epididymitis (C) Development of prostatitis (D) Formation of struvite kidney stones (E) Rapid dissemination throughout the body

*The answer is B.* This organism fits the description due to its involvement in struvite kidney stone production and because it is a highly motile, urease-producing, Gram-negative rod. Although U. urealyticum and P. aeruginosa have been reported to induce struvite stones, Proteus organisms are the most common cause.

A 78-year-old hospitalized male with an indwelling catheter develops a new onset of fever. The urine sample has an alkaline pH. The bacterium isolated from the urine is a urease-producing, highly motile, Gram-negative bacillus. What is the pathogen most likely involved in the above case? (A) Escherichia coli (B) Proteus mirabilis (C) Pseudomonas aeruginosa (D) Staphylococcus saprophyticus (E) Ureaplasma urealyticum

*The answer is C.* Escherichia coli is Lac+, and enteroinvasive strains characteristically cause a dysentery-like syndrome. Shigella characteristically causes bloody diarrhea (dysentery) but is Lac-. Pseudomonas aeruginosa characteristically causes infections in compromised hosts and is Lac-. Salmonella is also Lac-. Helicobacter pylori causes gastritis.

A Lac+, glucose-fermenting, gram-negative rod isolated from a previously healthy child with bloody diarrhea is most likely to be: A. Shigella sonnei. B. Pseudomonas aeruginosa. C. Escherichia coli. D. Salmonella enterica. E. Helicobacter pylori.

*The answer is D.* There are six serotypes of Haemophilus influenzae (a-f); capsular type b is the most invasive strain of H. influenzae and can cause sepsis, meningitis, pneumonia and other diseases. Additionally, there are unencapsulated strains referred to as nontypable Haemophilus influenzae because serotyping is based on antigens in the polysaccharide capsule. From the time that the H. influenzae type b (Hib) proteinpolysaccharide conjugate vaccine became available in 1987 for childhood immunization beginning at 2 months of age, there has been a dramatic decrease in the incidence of invasive disease caused by Haemophilus influenzae type b. immunity to this and other infectious disease is accomplished during the first months of life by lgG antibodies acquired transplacentally from the mother, but this protection is only transient. H. influenzae type b conjugate vaccines prevent disease by the induction of active B-lymphocyte mediated humoral immunity and may decrease oropharyngeal carriage of H. influenzae type b. Before the availability of the vaccine, slightly less than 50% of all cases of acute bacterial meningitis in the U.S. were caused by Haemophilus influenzae type b.

A developing country includes infant vaccination with the Haemophilus influenzae type b conjugate vaccine into its routine immunization schedule. If effectively implemented, this change in the vaccination schedule would most likely affect the epidemiology of: A. Otitis media B. Sinusitis C. Acute bronchitis D. Meningitis E. Conjunctivitis

*The answer is E.* All five organisms are opportunists capable of causing pneumonia in compromised patients. However, the first four are members of the family Enterobacteriaceae and, by definition, can ferment carbohydrates. In addition, none of these organisms are known to produce a green pigment, although Serratia may produce a red pigment. Pseudomonas aeruginosa is an obligate aerobe that uses respiratory pathways exclusively. Production of green pyocyanin pigment regularly occurs.

A male older adult, hospitalized and recovering from cardiac bypass surgery, develops pneumonia. Sputum culture reveals a gram-negative rod that produces a green pigment but does not ferment carbohydrates. The most likely organism is: A. Klebsiella pneumoniae. B. Serratia species. C. Proteus species. D. Enterobacter species. E. Pseudomonas aeruginosa.

*The answer is D.* Helicobacter pylori was first recognized as a possible cause of gastritis and peptic ulcer by Marshall and Warren in 1984. This organism is readily isolated from gastric biopsies but not from stomach contents. It is similar to Campylobacter species and grows on chocolate agar at 98.6°F (37"C) in the same microaerophilic environment suitable for C. Jejuni Campy-Pak or anaerobic jar [Gas Pak] without the catalyst). Helicobacter pylori, however, grows more slowly than C. Jejuni, requiring 5 to 7 days incubation. C jejuni grows optimally at 107.6℉ (42°C), not 98.6°F (37"C), as does H. Pylori

A patient with a burning epigastric pain is admitted to the hospital, and a gastric biopsy is performed. The tissue is cultured on chocolate agar incubated in a microaerophilic environment at 98.6°F (37°C) for 5 to 7 days. On fifth day of incubation, colonies appear on the plate and curved, gram-negative, oxidase-positive rods are observed. Which of the following is the most likely identity of this organism? a. Campylobacter fetus b. Campylobacter jejuni c. Haemophilus influenzae d. Helicobacter pylori e. Vibrio parahaemolyticus

*The answer is C.* While chemical analysis of the chloride content of the sweat is the most reliable and preferred diagnostic tool for a cystic fibrosis (CF), isolation of a mucoid variant of P. aeruginosa is considered pathognomonic for the disease. P. aeruginosa is the pathogen most frequently isolated from the respiratory tracts of pediatric and adult patients with CF. Burkholderia cepacia is another opportunistic pseudomonad that can be isolated from CF patients, but its prevalence is significantly less. COPD needs time to develop and would not be expected in a child. The child does not present with typical signs of pneumonia. Sarcoidosis is a systemic disease that is often manifested by a granulomatous infl ammation of the lungs. The etiology is unknown, and it is usually diagnosed in the adult patient. Bronchiolitis, a respiratory illness in children, is associated with the respiratory syncytial virus (RSV).

A pediatric patient with a persistent cough is evaluated for sinopulmonary disease. History and physical examination determines that the child suffers from nasal polyps and, possibly, chronic airway obstruction. A sputum sample produces several bacterial species, including Haemophilus influenzae and a mucoid variety of Pseudomonas aeruginosa. Which of the following is the most likely diagnosis? (A) Bronchiolitis (B) Chronic obstructive pulmonary disease (COPD) (C) Cystic fibrosis (D) Interstitial pneumonia (E) Sarcoidosis

*The answer is D.* The child in the case has meningitis. Organisms that cause meningitis in a child of this age include Streptococcus pneumoniae, Neisseria meningitides, and H. infl uenzae b. No other organism shares the culture characteristics of Haemophilus. This group of organisms is nutritionally fastidious and able to grow on chocolate agar (which supplies necessary nutrients from lysed red blood cells) or as satellite colonies surrounding β-hemolytic colonies of S. aureus on blood agar.

An 18-month-old male presents with fever, lethargy, malaise, productive coughing, and vomiting. Culture of cerebral spinal fluid samples with Staphylococcus aureus on blood agar reveals the presence of "satellite" colonies. Serology has indicated that the organism is polyribitol phosphate (PRP) positive. Gram stain of the colonies indicates the presence of a Gram-negative coccobacillus. Which one of the following organisms is most likely responsible for the disease? (A) Bordetella pertussis (B) Campylobacter jejuni (C) Escherichia coli (D) Haemophilus influenzae b (E) Pasteurella multocida

*The answer is A.* Klebsiella is a nonmotile, Gram-negative rod with a prominent polysaccharide capsule or K antigen. The capsule is the main determinant of pathogenicity, preventing phagocytosis of the organism unless it has been opsonized by complement or IgG. Other virulence factors include lipopolysaccharide or O antigen and siderophores involved in uptake of iron from the host.

An 85-year-old male nursing home patient with a history of alcoholism suddenly developed a flu-like illness. He complained of chills and fever and had frequent coughing spells productive of thick, bloody sputum. The attending physician diagnosed bronchopneumonia and prescribed antibiotics, but regrettably the patient died within a week. A bacterium, cultured on MacConkey agar (shown in the photograph), was isolated from sputum and blood of the patient in the above case. What is the primary function of the pathogenicity determinant depicted in this photo? (A) Antiphagocytic, unless opsonization occurs (B) Degrades secretory IgA on mucosal surfaces (C) Inhibits the function of complement (D) Lyses neutrophils and macrophages (E) Protease activity and disrupts membranes

*The answer is B.* All of the organisms listed are causes of diarrheal diseases; however, of the species listed, only E. coli O157:H7 is associated with this constellation of symptoms.

Hemorrhagic colitis, hemolytic uremic syndrome, and thrombotic thrombocytopenic purpura are clinical manifestations of which bacterial species? (A) Campylobacter jejuni (B) Escherichia coli O157:H7 (C) Shigella flexneri (D) Vibrio cholerae (E) Yersinia enterocolitica

*The answer is C.* Of the choices, only the chlamydiae and the rickettsia are obligate intracellular organisms. Chlamydiae are spread by direct contact or respiratory droplets, while the rickettsia (the correct answer) are spread commonly by arthropod vectors. Spirochetes like B. burgdorferi are not obligate intracellular pathogens and most are also not transmitted by arthropod vectors.

In any case, a clue indicating that the causative organism is an obligate intracellular pathogen transmitted by an arthropod bite should lead you to which of the following groups of organisms? (A) Chlamydiae (B) Enterobacteriaceae (C) Rickettsias including Anaplasma and Ehrlichia (D) Spirochetes including Borrelia burgdorferi

*The answer is D.* Most transmission in the United States is from an infected flea bite (a choice not given in the question). The other route of transmission is through respiratory droplets from patients who have developed pneumonic emboli and pneumonia.

Yersinia pestis may be transferred by (A) Dermacentor tick bite (B) Human body louse bite (C) Ixodes tick bite (D) Respiratory droplets

*The answer is A.* All of the agents listed can cause meningitis. The most common causes of meningitis in infants less than 1 month of age are E. coli K1, Group B Streptococcus, and Listeria monocytogenes. The clue to the identifi cation of E. coli as the etiologic agent is the fact that the organism grew on MacConkey agar—a selective medium for the growth of enteric, Gram-negative rods.

A bacterium was isolated from the central nervous system of a newborn that died of meningitis. The vagina of the mother was colonized with the same isolate as determined by the capsular antigen. The microbe grew well on standard blood and MacConkey agar at ambient atmospheric conditions. What is the most likely etiologic agent? (A) Escherichia coli K1 (B) Group B Streptococcus type III capsule (C) Haemophilus influenzae type b (D) Neisseria meningitidis serogroup b (E) Streptococcus pneumoniae (encapsulated variety)

*The answer is A.* The neonate in the above vignette suffers from meningitis secondary to an E. coil infection. E. coil is a motile gram- negative bacillus that is facultatively anaerobic and able to ferment both lactose and glucose. It grows well on blood, MacConkey, and eosin methylene blue (EMB) agar plates. MacConkey agar is a selective and differential medium used to isolate gram-negative organisms from contaminants in clinical specimens. When an organism ferments lactose on MacConkey agar, the local drop in pH causes the colony to take on a pink-red appearance. The bile salts present in MacConkey agar prevent the growth of Gram-positive organisms; therefore any CSF culture showing growth on MacConkey agar implies CNS infection with enteric bacteria—specifically, infection by E. coil. Group B Streptococcus is the most common cause of neonatal meningitis in the United States, followed by E. coil Listeria monocytogenes, and Klebsiella pneumoniae. Hemophilus influences type b also remains an important cause of meningitis in non immunized infants. The K-1 capsular antigen is present in 20-40% of intestinal E. coil isolates. E. coil can invade the blood stream of infants from the nasopharynx or GI tract and can then travel hematogenously to the meninges. The K-i antigen is considered the major determinant of virulence among strains of E. coil that cause neonatal meningitis. The K-1 antigen inhibits complement phagocytosis, and other host responses. The capsule is immunogenic and anti capsular antibodies are protective.

CSF cultures from a neonate with fever and poor feeding reveal motile Gram-negative rods that form pink colonies on MacConkey agar. Which of the following is the most important bacterial factor in the development of this infection? A. Capsule B. Verotoxin C. Antigen D. Fimbrial antigen E. Lipid A

*The answer is B.* P-fimbriae and type-1 fimbriae are significant virulence determinants of uropathogenic strains of Escherichia coli, the most common cause of urinary tract infections. These fimbriae mediate attachment of uropathogenic strains to uroepithelial cells. E. coli, which grows on MacConkey agar and ferments lactose, accounts for 70% to 90% of all the uncomplicated UTIs.

During a gynecologic office visit, a positive leukocyte esterase test of the urine along with a voiding symptom prompts the physician to order a urine culture. The bacterium, which was isolated in clinically significant numbers, grows profusely on MacConkey agar, as shown in the accompanying photograph, and ferments lactose. Which of the following cellular attributes is most important in determining virulence and classifying this microbe as a uropathogen? (A) β-lactamase (B) Fimbriae (C) Flagella (D) Glycocalyx (E) Urease

*The answer is C.* Francisella tularensis has a broad host range, including wild and domestic mammals, birds, and house pets. Pasteurella multocida primarily colonizes mammals and birds, both domestic and feral. Brucella melitensis primarily infects sheep and goats. Yersinia pestis infects a variety of mammals.

For which of the following organisms is there no known animal reservoir? A. Francisella tularensis B. Pasteurella multocida C. Bordetella pertussis D. Brucella melitensis E. Yersinia pestis

*The answer is A.* The "bull's-eye" appearance of the skin lesion, the fever, muscle and joint pain, and the fact that the patient lives in an endemic area support a clinical diagnosis of tickborne Lyme disease caused by B. burgdorferi.

During the summer in western Massachusetts, a teenager complains of aching muscles and joints and mild fever. He notices a rash on the side of his upper leg (shown in the photograph), which prompts his mother to take him to the clinic. What is the most likely etiology and route of transmission? (A) Borrelia burgdorferi/tick bite (B) Ehrlichia chaffeensis/tick bite (C) Francisella tularensis/deer fl y bite (D) Leptospira interrogans/rodent bite (E) Necrotic arachnidism/spider bite

*The answer is A.* Shiga-like toxins (Stx) or Vero cytotoxins (VT) are produced by Enterohemorrhagic E. coli (EHEC). Shiga-like toxins (Stx) or Vero cytotoxins (VT) are synonymous; the name Vero cytotoxin results from this toxin's ability to kill Vero cells in cell cultures. Shiga-like toxin I (Stx-l) or Vero cytotoxin (VT1) is the substance mentioned in the question stem that is able to inhibit protein synthesis in human cells, and it is closely related to the Shiga toxin produced by Shigella dysenteriae. Shiga-like toxin is an AB toxin that consists of an active A subunit and five binding B subunits. The plasmid coding for this toxin is transmitted to E. coli by a temperate bacteriophage. The B subunits form pentamers with a central pore. The B subunits recognize the specific receptors on the target cells and induce receptor-mediated endocytosis and internalization of the toxin. Subsequently, the enzymatically active A subunit is released. The A subunit then catalyses the removal of a single specific adenine residue there by preventing binding of tRNA to the 60S ribosomal subunit and inhibiting protein synthesis. This leads to intestinal mucosal cell death as well as direct toxicity to renal endothelial cells.

E. coli strains isolated from a 4-year-old Caucasian female with bloody diarrhea produce a substance that inhibits protein synthesis in human cells. The substance shares many properties with the toxin produced by: A. Shigella dysenteriae B. Salmonella typhi C. Pseudomonas aeruginosa D. Proteus mirabilis E. Vibrio cholerae F. Clostridium difficile

*The answer is C.* Haemophilus influenzae is a blood loving organism and requires both X factor (exogenous hematin) and V factor (NAD+) to support growth. Furthermore, 5% sheep blood agar plates not only lack sufficient nutrients to support the growth of Haemophilus species, but they also do not allow the growth of Haemophilus species because of the presence of V factor inactivating enzymes found in the media. Growth of Haemophilus species can be achieved on 5% sheep blood agar by cross streaking the medium with Staphylococcus aureus. Colonies of H. influenzae will grow around the hemolytic S. aureus coon resulting in the characteristic "Satellite" phenomenon. When the enzymes of beta hemolytic S. aureus lyse the red blood cells in the medium X factor (hematin) is released, and V factor (NAD+, choice C) is actively secreted by staphylococci into the growth medium. S. aureus thereby provides the X and V factors necessary to support the growth of Haemophilus species in sheep blood agar. Because Haemophilus species are part of the normal flora of the upper respiratory tract, it is critical to differentiate it from other bacteria through biochemical means. Part of the identification criteria is to demonstrate that colonies suspected of being Haemophilus species require X and V factor for growth.

Gram-negative coccobacilli isolated from the CSF of an infant with meningitis demonstrate little growth on sheep agar but grow well once incubated on the same media with Staphylococcus aureus. Staphylococci promote the growth of the bacteria by supplementing: A. ATP B. Catalase C. NAD+ D. Pyruvate E. Iron

*The answer is E.* Antibiotic treatment decreases the duration of illness and reduces person-to-person spread. Suitable antibiotics include ampicillin and TMP-SMZ unless resistance is noted. If antibiotic susceptibility is unknown, parenteral ceftriaxone sodium, a fluoroquinolone, or azithromycin is the drug of choice. Improved community sanitation and personal hygiene should also be encouraged. Antimotility drugs are contraindicated for patients with bloody diarrhea as they may prolong the duration of fecal shedding of the organism and increase the risk of bacteremia. Humans are the only host for Shigella; the organisms have no animal reservoir.

Medical outreach physicians, working in a refugee camp in central Africa, are concerned with the number of young children presenting with high fever, bloody diarrhea, and dehydration. The crowded living conditions of the camp suggest person-to-person contact, but because of the remoteness of the site only limited laboratory studies are available. The suspected culprit is described as a nonmotile, Gram-negative bacillus that does not produce lactose-positive colonies on selective media. A major virulence factor of the etiologic agent in the above question is a toxin. What is the proper course of action the physicians should take with regard to the management of the outbreak described in the above case? (A) Advise patients to increase their vitamin intake due to the effects of diarrhea (B) Avoid antibiotics and allow the symptoms to abate naturally (C) Avoid contact with animal reservoirs to prevent reintroduction of the organism into the community (D) Prescribe antimotility drugs to reduce transmission (E) Treat affected patients with antibiotics

*The answer is B.* Shiga toxin is an AB toxin; the gene for which is carried on the chromosome of Shigella dysenteriae. The B portion of the toxin binds to the receptors on intestinal cells and the A subunit enters the cells and cleaves the 60 S subunit of the ribosome, preventing the binding of transfer RNA, thereby stopping protein synthesis. Intestinal cells die, leading to bloody diarrhea and facilitating the invasion of the organism into the lamina propria.

Medical outreach physicians, working in a refugee camp in central Africa, are concerned with the number of young children presenting with high fever, bloody diarrhea, and dehydration. The crowded living conditions of the camp suggest person-to-person contact, but because of the remoteness of the site only limited laboratory studies are available. The suspected culprit is described as a nonmotile, Gram-negative bacillus that does not produce lactose-positive colonies on selective media. A major virulence factor of the etiologic agent in the above question is a toxin. What is the role of this toxin in disease production? (A) Acts as a superantigen thereby inducing inflammation (B) Blocks protein synthesis causing intestinal cell death (C) Increases cAMP concentration within intestinal cells (D) Increases cGMP concentration within intestinal cells (E) Suppresses cytokine secretion by immune cells

*The answer is B.* Syphilis, like Lyme disease, has three distinct phases. The other listed properties are either irrelevant or false. Unlike Treponema pallidum, Borrelia burgdorferi can be cultured, but the procedure is difficult and takes 6 to 8 weeks.

Syphilis and Lyme disease are strikingly similar in which of the following aspects? A. Their modes of transmission are similar. B. Both diseases display three similar, distinct phases. C. Both causative agents share many antigenic markers. D. The diseases show cross-immunity. E. Both causative agents can be cultured.

*The answer is C.* Any of these mechanisms are conceivable with various degrees of plausibility. The weight of evidence, however, favors the mechanism whereby new antigenic variants arise that elude, for a period, the host's immune defenses, to then be replaced by another variant.

The probable cause for the relapsing nature of relapsing fever caused by Borrelia recurrentis is: A. the sequential appearance of new antibiotic resistant variants. B. periodic spore dormancy and activation. C. successive appearance of antigenic variants. D. periodic hormonal fluctuations in the host. E. organisms that survive and propagate after spirochete-induced fever.

*The answer is D.*Coxiella burnetii is a rickettsia-like organism that can be spread via amniotic fluid, aerosols, or dust particles. It withstands drying and thus can be transmitted at least 10 miles by the wind.

A 23-year-old man who has recently started working on a sheep farm in Nova Scotia develops pneumonia shortly after helping with lambing. His cough produces little sputum, and a saline-induced sputum sample shows no predominant organism either with Gram stain or with acid-fast stain. It is established that he acquired the pneumonia from parturition products from the sheep. Which agent is most likely to be the cause of his pneumonia? (A) Rickettsia akari (B) Rickettsia typhi (C) Rickettsia rickettsii (D) Coxiella burnetii (E) Anaplasma phagocytophilum

*The answer is C.* Both pertactin and filamentous hemagglutinin mediate binding to the ciliated epithelial cells while tracheal cytotoxin, a peptidoglycan like molecule, specifi ally kills ciliated respiratory epithelial cells. M protein is an adhesive structure found on group A streptococci; pneumolysin is a virulence factor of Streptococcus pneumoniae which may kill ciliated epithelial cells; P-pili are adhesive structures of uropathogenic E. coli; and lecithinase is secreted by Clostridium perfringens. While capsular polysaccharide can promote adhesion, B. pertussis is not encapsulated. Polyribitol phosphate capsule is characteristic of Haemophilus influenzae and colonization factor is a virulence factor of Vibrio cholerae.

A 10-month-old male child presents with episodes of repetitive coughing with intermittent large gasps of air as well as some vomiting. Parents indicate that the child has been suffering from this condition for about 1 week. Incidentally, the previous week he was reported to have a cold-like illness with a fever and sneezing. A white blood cell count shows 65% lymphocytes and 30% neutrophils. An oxidase-positive, Gram-negative coccobacillus is grown from a nasopharyngeal swab plated on Regan-Lowe charcoal agar. What are the virulence factors of the organism in the above question that enable it to adhere to and kill ciliated respiratory epithelial cells? (A) Capsular polysaccharide and diphtheria toxin (B) M protein and pneumolysin (C) Pertactin, filamentous hemagglutinin, and tracheal cytotoxin (D) P-pili and lecithinase (E) Polyribitol phosphate capsule and colonization factor

*The answer is A.* The case fits the description of whooping cough or pertussis. This disease is characterized by repetitive bouts of unrelenting coughing punctuated by gasps of air and often end in vomiting. A whooping sound is often made when patients gasp for air. Lymphocytosis, at times as high as 70% of the peripheral white blood cell count, is typical for this disease. The causative agent is Bordetella pertussis, a fastidious organism that can be cultured on Regan-Lowe charcoal agar. While the other organisms listed cause respiratory disease, they are not associated with the disease described or microbiologic characteristics of the organism causing this case.

A 10-month-old male child presents with episodes of repetitive coughing with intermittent large gasps of air as well as some vomiting. Parents indicate that the child has been suffering from this condition for about 1 week. Incidentally, the previous week he was reported to have a cold-like illness with a fever and sneezing. A white blood cell count shows 65% lymphocytes and 30% neutrophils. An oxidase-positive, Gram-negative coccobacillus is grown from a nasopharyngeal swab plated on Regan-Lowe charcoal agar. Which one of the following organisms is most likely responsible for this disease? (A) Bordetella pertussis (B) Corynebacterium diphtheriae (C) Haemophilus influenzae (D) Mycoplasma pneumoniae (E) Streptococcus pneumoniae

*The answer is B.* Although Haemophilus influenzae type b is rarely seen in vaccinated children younger than 2 years of age, it still occurs in unvaccinated children. The description of a chocolate agar-positive organism should suggest either Neisseria meningitidis or H. influenzae type b. The descriptor as a short rod (from CSF) or pleomorphic rod (from culture) suggests H. influenzae type b. Although both Klebsiella pneumonia and Escherichia coli (a cause of neonatal meningitis) are both Gram-negative rods, neither is likely to cause meningitis in a healthy child, and, like all other Enterobacteriaceae, will grow on blood agar. Streptococcus is Gram-positive.

A 14-month-old boy is brought in by his parents with fever, fussiness and lethargy, and apparent headache. On examination, the neck is stiff. His parents have not allowed his routine childhood vaccines. Very short Gram-negative rods are seen in the CSF, so antibiotics are immediately started. The organism grows on chocolate agar but not blood agar. No one else in the family is ill. What is the most likely causative agent? (A) Escherichia coli (B) Haemophilus influenzae type b (C) Klebsiella pneumoniae (D) Neisseria meningitidis (E) Streptococcus pneumoniae

*The answer is A.* All the organisms listed cause diarrhea; however, only C. jejuni and Salmonella sp. typically cause bloody diarrhea. Shigella flexneri causes bloody diarrhea whereas S. sonnei typically causes a watery diarrhea. Of the organisms listed, only C. jejuni is a microaerophilic (requiring reduced oxygen in the environment to grow), Gram-negative, curved rod that demonstrates darting motility when viewed under a microscope.

A 15-year-old boy presents with diarrhea, malaise, fever, and abdominal pain. The stools were positive for occult blood and fecal leukocytes. Microscopy of the stool also reveals the presence of curved bacilli that moved about on a slide in a darting motion. The etiologic agent, a Gram-negative, curved rod, grew on specialized medium in microaerophilic conditions. Which one of the following organisms is most likely responsible for the disease? (A) Campylobacter jejuni (B) Enterotoxigenic Escherichia coli (C) Salmonella sp. (D) Shigella sonnei (E) Vibrio cholerae

*The answer is E.* The test described is a screening assay for the presence of urease activity, an indirect means of detecting the presence of Helicobacter pylori, a major cause of duodenal ulcer. Culturing the organism from gastric biopsy specimens is considered the definitive confirmatory test. The noninvasive urease breath test involves consuming a solution containing isotopically-labeled urea. When present urease (a product of H. pylon) degrades the urea into carbon dioxide and ammonia. The isotopically-labeled carbon dioxide is absorbed into the bloodstream and exhaled in the patient's breath. Typically, breath samples are collected 30 minutes after the labeled urea is ingested. This test has excellent sensitivity and specifically for both the initial diagnosis of H. pylori infection and for monitoring treatment success. Antibiotic or proton pump inhibitor use during the 2-4 weeks prior to the test may 4cause false negative results.

A 22-year-old male ingests a solution containing 13C-labeled urea. He is then asked to blow into a tube, and the labeled carbon is detected in his breath samples. This test is most likely part of the evaluation for which of the following conditions? A. Bronchiectasis B. Pulmonary fibrosis C. Congenital heart disease D. Chronic hepatitis E. Duodenal ulcer F. Acute pancreatitis G. Chronic diarrhea

*The answer is C.* Staphylococcus aureus and Pseudomonas aeruginosa are two primary pulmonary colonizers that cause pneumonia in patients with cystic fibrosis. (Staphylococcus is usually only in young CF patients.) Of the two, Pseudomonas is Gram negative. Its slime material (alginate) produces the resistance to phagocytic killing and poor penetration of antibiotics to the site, which, in conjunction with the antibiotic resistance of Pseudomonas, make these serious infections.

A 22-year-old man with cystic fibrosis presents with fever and increasing dyspnea. A Gram-negative organism is found in unusually high numbers in the pulmonary mucus. Which virulence factor is most important in colonization and maintenance of the organism in the lungs? (A) Exotoxin A (B) Pyocyanin (blue-green pigment) (C) Polysaccharide slime (D) Endotoxin

*The answer is M.* Usually, the urinary tract and urine are sterile. The presence of microorganisms in the urine is due to either a urinary tract infection (UTI), contamination or both. Contamination is highly suspected if the urine is polymicrobial or atypical of the microbial flora commonly seen in UTIs. A UTI can be symptomatic (cystitis, pyelonephritis) or asymptomatic (asymptomatic bacteriuria). It affects an estimated 20% of women at some time during their lifetimes. Most often the infection is due to invasion by enteric coliform bacteria that inhabit the periurethral vaginal introitus and ascend into the bladder via the urethra. The most common causative organism is Escherichia coil, which accounts for 80% or more of cases. Most of the remainder of cases in sexually active women are caused by Staphylococcus saprophy1/cus other gram-negative enterics such as Proteus account for a smaller percentage. Normally, symptomatic UTIs are treated with a three-day course of a fluoroquinolone or TMP-SMX (trimethoprim sulfamethoxazole). Symptoms of fever chills and flank pain should alert the physician to the possibility of pyelonephritis.

A 24-year-old female presents to your office with burning during urination, urine clouding and urinary frequency. She denies fever, chills and flank pain. She had a similar episode before that was treated with antibiotics. She has no other medical problems and does not use tobacco alcohol or drugs. Her vital signs are stable. Physical examination shows suprapubic tenderness. Which of the following bacteria is most likely to be isolated from this patient's urine? A. Klebsiella pneumoniae B. Salmonella typhi C. Salmonella enteritidis D. Campylobacter jejuni E. Shigella dysenteriae F. Proteus mirabilis G. Yersinia enterocolitica H. Haemophilus Influenzae I. Vibrio cholerae J. Streptococcus pyogenes K. Streptococcus pneumoniae L. Entamoeba histolytica M. Escherichia coli

*The answer is D.* All of the organisms listed are Gram-negative rods; however, only P. aeruginosa fits the microbiologic description. It is oxidase-positive, a test often used to distinguish this species from other Gram-negative bacteria such as the other organisms listed. The photograph demonstrates the characteristic pigment production by P. aeruginosa. This organism produces two pigments, pyocyanin and pyoverdin, which are important virulence factors. P. aeruginosa is an important nosocomial pathogen resistant to many antibiotics. In the hospital, it is an important cause of burn wound infections as well as urinary tract infections, pneumonia, and bacteremia.

A 27-year-old male was hospitalized suffering third-degree burns over 50% of his body from a house fire. After 6 days in the hospital, he becomes septic and the burn wounds show tissue necrosis in several areas. Blood and wound cultures grew an oxidase-positive, Gram-negative bacillus resistant to aminopenicillins, macrolides, and first-generation and second-generation cephalosporins. The organism growing on nutrient agar is shown in the photograph. What is the etiology of this man's burn infection? (A) Enterobacter cloacae (B) Klebsiella pneumoniae (C) Proteus vulgaris (D) Pseudomonas aeruginosa (E) Serratia marcescens

*The answer is D.* The case is descriptive of hemolytic uremic syndrome associated with infection with E. coli O157:H7. This strain of E. coli produces phage-associated Shiga toxins (Stx-1 and/or Stx-2) that are functionally and antigenically similar to that produced by Shigella dysenteriae. None of the other organisms listed are associated with hemolytic uremic syndrome, although some do cause a bloody diarrhea. C. perfringens is associated with ingestion of contaminated meats but it causes watery diarrhea. S. typhi, rare in the United States, causes typhoid fever. Enteroinvasive E. coli strains are uncommon causes of diarrhea. Y. enterocolitica, also rare in the United States, causes a bloody diarrhea.

A 3-year-old boy developed bloody diarrhea after eating a poorly cooked hamburger. A few days later, he was hospitalized with oliguria and hypertension. With which organism is he most likely infected? (A) Clostridium perfringens (B) Salmonella typhi (C) Enteroinvasive E. coli (D) Shiga toxin-producing E. coli (E) Yersinia enterocolitica

*The answer is D.* This self-limited diarrhea "traveler's diarrhea" is characteristic of Enterotoxigenic E. coli (ETEC). E. coli are gram- negative motile enteric rods that normally colonize the human GI tract. The usual presentation of ETEC gastroenteritis is watery diarrhea with abdominal cramping, nausea and vomiting, and possibly a low fever. ETEC causes diarrhea in infants and travelers in underdeveloped countries or regions with poor sanitation. The disease requires colonization by the ETEC, usually in the small intestine, and subsequent elaboration of the heat labile (LT) and! or heat stable (ST) enterotoxins. Both of these toxins are plasmid-encoded. The LT enterotoxin is very similar to cholera toxin in both structure and mode of action. LT increases intracellular cAMP in gut mucosal cells by activating the stimulatory Gs membrane G protein thus activating adenylate cyclase. The ST enterotoxin is not inactivated by heat because of its small molecular size. ST causes an increase in cyclic GMP in the host cell cytoplasm by binding to and activating guanylate cyclase located on the apical membranes of host gut mucosal cells. The action of both the LT and the ST enterotoxins is increased secretion of fluid and electrolytes resulting in watery diarrhea. Fimbriae (pili) are responsible for bacterial adhesion to the gut mucosa.

A 32-year-old Caucasian male develops profuse watery diarrhea with abdominal cramps, nausea and vomiting during a trip to Mexico. The diarrhea resolved in a couple of days without antibiotic treatment. This patient's symptoms are most likely related to: A. Intestinal invasion B. Stacked-brick intestinal adhesion C. Shiga-like toxin D. Cholera-like toxin E. Endotoxin release

*The answer is E.* Epiglottitis is almost exclusively caused by Herophilus influenzae type b and was commonly seen in children between 2 and 7 years old prior to introduction of the polysaccharide-protein conjugate Hib vaccine in the late 1980s. It was also the common cause of meningitis prior to the vaccination. Epiglottitis is usually an acute infection, with an abrupt onset of obstructive laryngeal edema. Diagnosis can be made on symptoms of acute fever inspiratory stridor, drooling odynophagia, and positive "thumb sign" on lateral cervical x-ray due to the edematous epiglottis. The capsule of H. influenzae is immunogenic and is composed of a linear polymer composed of ribose, ribitol, and phosphate referred to as polyribosyl-ribitol-phosphate (PRP). Antibodies to PRP facilitate complement dependent phagocytosis and killing thru opsonization. To effectively prevent infection with H. influenzae type b individuals must mount an antibody response against the capsular material. In children less than 2 months old humoral immunity to H. influenzae is received transplacentally from maternal lgG if the mother is immune. The first dose of H. influenzae type b vaccine is given at 2 months of age.

A 5-year-old boy is found to have a high serum level of antibodies against polyribitol ribose phosphate (PRP). The antibodies would most likely carry protection against A. Pyelonephritis B. Miliary tuberculosis C. Rheumatic fever D. Osteomyelitis E. Epiglottitis F. Malignant pustule

*The answer is A.* Acute pyelonephritis is an infectious disease involving the kidney parenchyma and the renal pelvis. Gram-negative bacteria, such as Escherichia coli, Proteus, Klebsiella, and Enterobacter, are the most common causative organisms in acute pyelonephritis. Laboratory evaluation will often reveal leukocytosis with a left shift, and urinalysis typically shows pyuria, varying degrees of hematuria, and white cell casts. Since bacteremia is present, the patient should be hospitalized and empirically started on IV ampicillin and gentamicin. This regimen may be need to be changed, however, once the sensitivity results are available. Erythromycin (choice B) and tetracycline (choice E) are both bacteriostatic antibiotics and would not be recommended in a patient with a severe infection, such as acute pyelonephritis with bacteremia. Vancomycin (choice C) is primarily used in the treatment of severe gram-positive infections. Phenazopyridine (choice D) is a urinary analgesic, and nitrofurantoin (choice D) is a urinary tract anti-infective. Although nitrofurantoin is indicated for the treatment of "mild" cases of pyelonephritis, as well as cystitis, this patient's condition is severe and should be treated with appropriate antibiotics.

A 33-year-old woman presents with fever, vomiting, severe irritative voiding symptoms, and pronounced costovertebral angle tenderness. Laboratory evaluation reveals leukocytosis with a left shift; blood cultures indicate bacteremia. Urinalysis shows pyuria, mild hematuria, and gram-negative bacteria. Which of the following drugs would best treat this patient's infection? A. Ampicillin and gentamicin B. Erythromycin C. Gentamicin and vancomycin D. Phenazopyridine and nitrofurantoin E. Tetracycline

*The answer is D.* E. coli is part of the normal bacterial flora of the intestinal tract. It is also a motile gram-negative bacillus that is facultatively anaerobic and able to ferment both lactose and glucose. E. coli grows well on blood MacConkey, and eosin methylene blue (EMS) agar plates. EMB agar is a selective and differential medium used to isolate and identity enteric pathogens from contaminated clinical specimens. On EMS agar, organisms that ferment lactose such as F. coli bind to dye in the agar and produce blue-black colonies that have a distinct metallic sheen. Most strains of E. coli demonstrate beta-hemolysis on blood agar. E. coli is responsible for urinary tract infections (UTI), neonatal meningitis, and several types of gastroenteritis. The virulence factors expressed by a particular strain of E. coli will determine disease. Adhesion is one of the most important virulence factors expressed by E. coli more specifically; fimbriae (Type 1 fimbriae, P fimbriae and S fimbriae) permit the adhesion of F. coli to uroepithelial cells (choice D). E. coli causes approximately 80% of all urinary tract infections. UTIs are most common in females, owing to the significantly shorter length of the female urethra. E. coli originate in the feces and spreads to colonize the perineal, genital, and—most importantly—the periurethral regions. During, for instance, sexual intercourse or bladder catheterization, E. coil can be propelled into the urethra and bladder from the colonized periurethral region. P fimbriae (pyelonephritis-associated pili) adhesion allow the uropathogenic F. coli to colonize the bladder and ureters, causing cystitis and pyelonephritis.

A 34-year-old female presents with fever, dysuria and flank pain; a urine culture grows motile Gram-negative rods in colonies that have a metallic sheen on EBM agar and demonstrate hemolysis on blood agar. Which of the following is the most important pathogenic bacterial factor responsible for this patient's condition? A. Capsule B. Heat-labile enterotoxin C. O antigen D. Fimbrial antigen E. Lipid A

*The answer is A.* This organism and nontyphoid strains of Salmonella are associated with ingestion of undercooked chicken or raw eggs. Both are common causes of foodborne infections in the United States and both cause a bloody diarrhea that is fecal leukocyte-positive. C. perfringens food poisoning is associated with ingestion of contaminated meat products including poultry. This organism causes an intoxication characterized by abdominal cramps and watery diarrhea occurring 8 to 24 h after ingestion. S. typhi and S. dysenteriae are strict human pathogens and are not associated with ingestion of undercooked chicken. Both are uncommon in the United States. Y. enterocolitica infection, also uncommon in the United States, is generally associated with ingestion of contaminated dairy products.

A 34-year-old woman presents with fever, nausea, severe abdominal cramping, and bloody diarrhea that was fecal leukocyte-positive. A few days previously, she had eaten undercooked chicken. Of the following, which is the most likely etiologic agent? (A) Campylobacter jejuni (B) Clostridium perfringens (C) Salmonella typhi (D) Shigella dysenteriae (E) Yersinia enterocolitica

*The answer is D.* The clinical syndrome described is consistent with "hot tub folliculitis," a superficial Pseudomonal infection of the hair follicle. The presentation of this condition is most commonly in outbreaks from public or hotel swimming pools or hot tubs where the chemicals in the pool water have not been maintained at the appropriate concentrations thereby allowing P. aeruginosa proliferation in the pool water. Many infections by P. aeruginosa often begin with exposure to a water source or creation of a moist environment (i.e. swimmer's ear, hot tub folliculitis, burn wound). Pseudomonas aeruginosa, a gram-negative rod, is ubiquitous in nature and is commonly isolated from water sources. Evidence indicating that the organism responsible for this patient's illness is Pseudomonas aeruginosa includes oxidase positivity which is indicative of the genus Pseudomonas, production of pigment during culture (pyocyanin, properdin), and the characteristic clinical picture of "hot tub folliculitis, an illness almost exclusively caused by P. aeruginosa.

A 35-year-old female presents to the ER with a diffuse pruritic papulopustular rash. Her friend was brought to the ER with similar symptoms two days ago. Pustular fluid demonstrates oxidase-positive gram-negative rods that produce pigment on culture medium. Which of the following is the most likely source of this patient's infection? A. Human contact B. Insects C. Food D. Water E. Soil F. Pets

*The answer is B.* Bacterial causes of watery diarrhea include Vibrio cholerae and enterotoxigenic Escherichia coli. Both of these organisms produce a toxin (cholera toxin and labile toxin, respectively) whose active portion acts to increase intracellular levels of cyclic AMP (cAMP). cAMP prevents sodium absorption into the intestinal cells and increases chloride secretion from the cells. Water moves from the cells into the lumen of the intestine in response to the movement of ions, thereby leading to watery diarrhea.

A 35-year-old male became ill with watery diarrhea due to a bacterial infection while on a trip to South America. Which of the following is the key event that promotes the development of watery diarrhea? (A) Decreased activity of adenylate cyclase (B) Increase in the intestinal epithelial cell levels of cAMP (C) Increase in the levels of intracellular chloride (D) Movement of water from the intestinal lumen into the epithelial cells. (E) Movement of ions such as potassium and sodium into the intestinal epithelial cells

*The answer is A.* Epiglottitis is the most common disease of the upper respiratory tract produced by Haemophilus influenzae type b, a gram negative encapsulated rod. It is also a common cause of otitis media in children and may cause bronchitis, bronchiolitis, and pneumonia in adults. The incidence of serious disease caused by Haemophilus influenzae type b has decreased greatly with the introduction of an effective vaccine. The vaccine is composed of the H. influenzae type b capsular polysaccharides coupled to a carrier molecule, given to children between 2 and 15 months of age. The patient had not received the Hib conjugate vaccine and therefore was susceptible to this organism. Klebsiella pneumoniae (choice B) causes pneumonia and pulmonary abscesses, but is not considered to be a pathogen in the upper respiratory tract. Legionella pneumophila (choice C) causes pneumonia in man. The disease may be mild (an atypical pneumonia) or a fulminating disease with a high mortality (30%). Mycoplasma pneumoniae (choice D) causes community-acquired atypical pneumonia. It is the most common cause of pneumonia in young adults. Streptococcus pyogenes (choice E) is the most common cause of pharyngitis; however, this patient's presentation strongly suggests epiglottitis.

A 4-year-old boy is brought to the emergency room in extreme respiratory distress, with a temperature of 103.8 degrees Fahrenheit. He is drooling and has difficulty swallowing, and on physical examination, inspiratory stridor is noted. A lateral x-ray shows swelling of the epiglottis. He has had no previous vaccinations. Which of the following agents is the most likely cause of these symptoms? A. Haemophilus influenzae B. Klebsiella pneumoniae C. Legionella pneumophila D. Mycoplasma pneumoniae E. Streptococcus pyogenes

*The answer is C.* H. influenzae is a gram-negative rod that can be either encapsulated or unencapsulated. Herophilus influenzae is a blood loving organism and requires both X factor (exogenous hematin) and V factor (NAD+) to support growth. There are six capsular serotypes classified as types a-f. Most infections are caused by H. influenzae strains belonging to capsular serotype b. Type b is the only H. influenzae capsular serotype that contains a ribose rather than hexose, as the carbohydrate component of the capsule, and this feature may be related to the virulence of the organism. The type b capsular material is a linear polymer consisting of ribose, ribitol and phosphate which is known as poly ribosyl-ribitol-phosphate (PRP). The PRP capsule prevents phagocytosis and intracellular killing by neutrophils and is essential for virulence of this organism (Choice C). Antibodies to serotype bare shown to promote opsonization, complement fixation. and phagocytosis and killing of H. influenzae type b. Unencapsulated (nontypable) H. influenzae are part of the normal flora of the upper respiratory tract. They rarely cause disease, and when they do it is only local infection such as sinusitis, otitis media or bronchitis; without a capsule they do not cause invasive disease.

A 4-year-old immigrant boy is brought to the pediatric emergency room with a swollen right knee accompanied by fever and malaise. He is hypotensive and tachycardic. His past medical history is nothing significant except for one episode of otitis media. Some of his vaccinations are not up-to-date. Synovial fluid and blood cultures grew pleomorphic gram negative rods on hematin containing medium. The pathogenesis of the organism responsible for his condition is most likely related to which of the following? A. Endotoxin B. Fimbriae C. Capsule D. Cytotoxic exotoxin E. Hemolysins F. Hyaluronidase

*The answer is C.* Tetracyclines are the drug of choice for rickettsial infections, and doxycycline is the preferred member of this group. Sulfonamides are contraindicated as they have been shown to worsen the symptoms.

A 40-year-old man presented with a rubeola-like rash on the extremities, chills, fever, myalgia, and malaise 5 days after returning from a June fishing trip in Arkansas. A history of tick bites is noted and Rocky Mountain spotted fever (RMSF) is suspected. With reference to the above question, which antimicrobial agent should be employed? (A) Chloramphenicol (B) Ciprofloxacin (C) Doxycycline (D) Penicillin G (E) TMP-SMZ

*The answer is E.* This enzyme neutralizes gastric acids. The organism also secretes catalase and superoxide dismutase which provide resistance to killing by neutrophils and mucinase that breaks down the mucus, allowing the organism to contact gastric epithelial cells. Another important virulence factor of this organism is vacuolating cytotoxin that leads to epithelial cell injury and neutrophil infiltration.

A 47-year-old male is diagnosed with peptic ulcer disease. Treatment includes antibiotics for infection of the stomach with Helicobacter pylori. Which enzyme is secreted by this organism and enables it to survive in the acid environment of the stomach? (A) Catalase (B) Oxidase (C) Protease (D) Transpeptidase (E) Urease

*The answer is B.* Acute epiglottitis is a rapidly progressive infection of the epiglottis leading to severe inflammation and edema of the epiglottis and larynx and potentially acute obstruction of the airway especially during laryngoscopy. Small children typically present with fever and dysphagia, while older children and adults complain of sore throat. Inspiratory stridor and anxiousness due to compromised diameter of the larynx occur frequently, and patients may present with drooling. Diagnosis is confirmed by the presence of an edematous epiglottis that classically appears cherry red though inspection of the epiglottis should not be done unless the team is prepared to provide a surgical airway by tracheostomy. The most likely pathogen in children is H. influenzae, but the introduction of the H. influenzae type b (Hib) vaccine has led to a dramatic decrease in the incidence of invasive disease caused by Haemophilus influenza type b including epiglottitis, meningitis, sepsis and other diseases commonly caused by this bacterium. The Haemophilus influenzae b vaccine is now a part of the routine vaccination schedule and is administered in the first few months of life. Immunity during the first couple months of life prior to this vaccination is acquired by transient maternal transplacental lgG antibodies. If a child were to miss a vaccination or not be vaccinated at all, then the child would be at an increased risk of H. influenzae type b infection, though Hib epiglottitis has been shown to occur in fully immunized children as well.

A 5-year-old boy is brought to the FR with breathing difficulty, dysphagia, drooling and fever. His temperature is 39.4C (103.0 F). White blood cell count is 23,000fcmm with many band forms. Laryngoscopy in the operating room shows a swollen and cherry-red epiglottis. This patient most likely: A. Traveled recently B. Missed vaccination C. Was stung by a bee D. Has penicillin allergy F. Has characteristic family history

*The answer is A.* This person has Lyme disease, which is caused by Borrelia burgdorferi.

A 52-year-old woman living in Maryland comes to the physician because of a 1-week history of low-grade fever, fatigue, and a red rash over the skin behind her left knee. She is an avid hiker. Her temperature is 38°C (100.4°F). Physical examination shows an 8-cm, warm, nontender, erythematous lesion with partial central clearing over the skin of the left popliteal area. Which of the following is the most likely causal organism? (A) Borrelia burgdorferi (B) Ehrlichia chaffeensis (C) Francisella tularensis (D) Rickettsia rickettsii (E) Streptobacillus moniliformis

*The answer is C.* While all of the organisms listed are associated with GI symptoms, this case is descriptive of peptic ulcer disease, which is associated with H. pylori infection. The other organisms cause diarrheal diseases.

A 52-year-old woman presented with indigestion and heartburn occurring shortly after meals which she treated with over-the-counter antacids. Physical examination revealed mild epigastric tenderness. A radiolabeled-urea breath test is positive. What is the most probable etiologic agent? (A) Campylobacter jejuni (B) Clostridium difficile (C) Helicobacter pylori (D) Shigella dysenteriae (E) Yersinia enterocolitica

*The answer is A.* his man has pneumonia caused by Klebsiella pneumoniae, as suggested by his alcoholism, lobar consolidation, and "currant jelly" sputum. Klebsiella is a member of the family Enterobacteriaceae, so it is a gram-negative, oxidase-negative bacillus. It typically affects the upper lobes of the lungs, although lower lobe involvement is not rare. A gram-negative, oxidase-positive bacillus (choice B) describes Pseudomonas aeruginosa, which may cause pneumonia but would be more likely in a patient with cystic fibrosis than in an alcoholic. The sputum would have a blue-green color and a fruity odor. A gram-negative pleomorphic bacillus requiring factors X and V (choice C) describes Haemophilus influenzae. It is a cause of pneumonia, pharyngitis, bronchitis, and otitis media, but does not typically cause lobar consolidation or bloody sputum. A gram-positive, catalase-negative coccus (choice D) describes the genus Streptococcus, of which S. pneumoniae is the most common cause of typical pneumonia in adults (including alcoholics). The sputum of streptococcal pneumonia, however, is "rusty," or tinged with brown blood, not bright red and gelatinous as described here. A gram-positive, catalase-positive coccus (choice E) describes the genus Staphylococcus, of which S. aureus is a common cause of pneumonia. It is most commonly associated with aspiration pneumonia in alcoholics, but causes a salmon-colored sputum, not one that is bright red and gelatinous.

A 58-year-old man is brought to the emergency department by police after being found staggering in the streets. His rectal temperature is 38.8ºC (101.8ºF), and he smells of wine. His coughing produces thick, bloody sputum. Physical examination shows dullness to percussion, increased vocal fremitus, decreased breath sounds, and wet crackles over the right lung fields. A chest x-ray shows consolidation of the right upper lobe. Which of the following is the most likely causal organism? A. A gram-negative, oxidase-negative bacillus B. A gram-negative, oxidase-positive bacillus C. A gram-negative, pleomorphic bacillus requiring factors X and V D. A gram-positive, catalase-negative coccus E. A gram-positive, catalase-positive coccus

*The answer is E.* This woman is suffering from septic shock. Septic shock results form the release of endotoxins into the bloodstream. Regardless of the bacterial source, most mammals, including humans experience the same range of toxic biological effects as a result of these endotoxins. Endotoxins are found in the outer membrane of Gram-negative bacteria which is composed of lipopolysaccharide (LPS). LPS is released during destruction of the bacterial cell wall. It can also be released during cell division. LPS is a very long heat-stable molecule arranged into three regions: O antigen core polysaccharide and Lipid A. Lipid A is responsible for the toxic properties of LPS that lead to Gram-negative sepsis and endotoxic septic shock. Lipid A induces shock by activation of macrophages and granulocytes. This activation results in the synthesis of endogenous pyrogens such as IL-1 prostaglandins, and the inflammatory mediators: tumor necrosis factor-alpha (TNF-alpha) and interferon. These cytokines then induce a febrile response by the action of IL-1 on the hypothalamus, as well as hypotension increased vascular permeability with third-spacing of fluids diarrhea disseminated intravascular coagulation and death.

A 59-year-old female is brought to the ER with fever skin flushing and an altered level of consciousness. Her blood pressure is 50/20 mmHg, and her heart rate is 120/mm. If blood cultures are positive for E. coli, which of the following bacterial factors is most likely responsible for this patient's current condition? A. Capsule B. Heat-stable exotoxin C. O antigen D. Fimbrial antigen E. Lipid A F. Flagellar antigen

*The answer is C.* This child has whooping cough, or, more accurately if one goes by the chronology of the symptoms, "coughing whoop," inasmuch as the patient is subject to a series of coughing episodes that are followed by a rapid inspiration of air. The "whoop" is caused by the rapid passage of air through a narrowed airway. The infection of the respiratory tract by Bordetella pertussis causes a hyperreactivity of the respiratory apparatus; even the slightest stimulus will trigger a coughing episode. Patients with whooping cough characteristically have posttussive emesis (vomit after a coughing episode). One of the toxins produced by B. pertussis is Pertussis toxin (A and B component), which ADP ribosylates Gi (thus inhibiting the negative regulator of adenylate cyclase) leading to increased cAMP. Multiple effects are produced by interrupting activities regulated by cAMP. As a result, it causes: Increased insulin production leading to hypoglycemia Lymphocytosis promotion Blockade of immune effector cells Increased histamine sensitivity

A 6-week-old girl is brought to the emergency department because of a 10-day history of coughing and choking spells. The child gasps for breath, experiences paroxysms of coughing during which her oxygen saturation drops to 86%, and vomits twice in the examination room. Laboratory studies show leukocytosis with 80% lymphocytes. Cultures of sputum on special media grow encapsulated, gram-negative rods. Which of the following additional findings is most likely to be seen? A. Erythematous rash B. Fibrinous pseudomembrane C. Hypoglycemia D. Koplik spots E. Vesicular rash

*The answer is D.* Proteus mirabilis is a urease-positive organism that displays swarming motility on nonselective agar.

A 69-year-old woman comes to the emergency department because of a 2-day history of increasingly severe fever and back pain; she also has a burning sensation with urination, and there is an aromatic smell to the urine. She has had three urinary tract infections treated with ciprofloxacin during the past year. Her temperature is 39.1°C (102.3°F). Physical examination shows right flank tenderness. Laboratory studies show: Leukocyte count 15,500/mm3 Urine pH 9 Protein trace WBC numerous Bacteria >100,000 colonies/mL A urine culture grows a urease-positive organism that displays swarming motility on nonselective agar. Which of the following is the most likely causal organism? (A) Enterococcus faecalis (B) Escherichia coli (C) Klebsiella pneumoniae (D) Proteus mirabilis (E) Staphylococcus saprophyticus

*The answer is E.* This Gramnegative rod usually causes a diarrheal illness but can also cause infections that mimic appendicitis. Y. enterocolitica is part of the normal intestinal flora of cattle and other farm animals. Human infection occurs by ingestion of food or water contaminated with animal feces. The organism is able to grow at refrigerated temperatures while other organisms present in the same fecal sample cannot. This isolation technique is known as cold enrichment. S. typhi and C. jejuni have been isolated from diseased appendices; however, these organisms are isolated at 37 °C. C. difficile is an anaerobic, Gram-positive rod causing antibiotic-associated diarrhea and pseudomembranous colitis. Vibrio parahaemolyticus, a curved, Gram-negative rod, causes a diarrheal illness following ingestion of inadequately cooked seafood.

A 7-year-old boy presents to the emergency department with fever, abdominal pain, and tenderness in the right lower quadrant. A white blood cell count reveals leukocytosis. Suspecting appendicitis, the child is taken to surgery; however, the removed appendix is grossly and histologically normal. The bacterial cause for the boy's symptoms was subsequently identified as a Gram-negative rod after cold enrichment of fecal cultures. What is the etiology of this infection? (A) Campylobacter jejuni (B) Clostridium diffi cile (C) Salmonella typhi (D) Vibrio parahaemolyticus (E) Yersinia enterocolitica

*The answer is A.* E. coli is the most common cause of UTI in any age group. The other organisms can cause UTI but are much less common.

A 7-year-old female child is taken to an urgent care facility because of fever and periodic incontinence. The physician suspects a urinary tract infection. What is the most likely etiology of this child's infection? (A) Escherichia coli (B) Proteus mirabilis (C) Pseudomonas aeruginosa (D) Staphylococcus saprophyticus (E) Ureaplasma urealyticum

*The answer is C.* This man is suffering from septic shock. The most common source of E. coli bacteremia is the urinary tract. Some common predisposing factors to urosepsis are urinary obstruction (i.e. BPH) fecal incontinence, a neurogenic bladder secondary to diabetes, and frequent or indwelling catheterization. Gram-negative sepsis or septic shock results from the body's systemic reaction to lipopolysaccharide endotoxin (a component in the membranes of some bacteria). Signs of sepsis include hyper or hypothermia tachycardia, an elevated respiratory rate, and an increased white blood cell count. Hyperventilation can lead to a respiratory alkalosis. Severe sepsis is marked by organ dysfunction from poor blood flow. Oliguria from poor renal perfusion and altered mental status from poor cerebral perfusion are often present. Late-stage sepsis will also manifest as metabolic lactic acidosis as a result of poor tissue perfusion. Treatment at this time is focused on maintaining blood pressure and organ perfusion with intravenous fluids, administering antibiotics specific for the offending organism, and maintaining proper ventilation and oxygenation.

A 72-year-old man is brought to the emergency department from a skilled nursing facility because of altered mental status, skin pushing, and a high fever. His blood pressure is 60/20 mmHg, and heart rate is 110/mm. If blood cultures are positive for E. coli, which of the following is the most likely source of bacteremia in this patient? A. Small intestine B. Colon C. Urinary tract D. Biliary tree E. Lungs F. Oropharynx G. Skin

*The answer is B.* The endoflagella are thought to be responsible for the corkscrew motion of spirochetes. The other structural features are not specific to spirochetes.

A distinctive feature of spirochetes is the presence of: A. fimbriae. B. endoflagella. C. helically arranged pili. D. nucleosomes. E. variable surface antigens.

*The answer is C.* This person is suffering from typhoid fever, caused by Salmonella enterica serovar typhi. Both enterohemorrhagic Escherichia coli and Shigella dysenteriae produce Shiga or Shiga-like toxins, which were not detected in this case. Both Helicobacter pylori and Campylobacter jejuni are curved organisms, which is not consistent with the cell morphology of the organism causing this infection. Moreover, neither H. pylori nor C. jejuni can be cultured on typical primary plating media such as MacConkey agar.

A young man returned from a backpacking trip in Mexico suffering from a high fever, pain in the abdomen, and watery diarrhea. The emergency room doctor noted a faint rash on the patient's abdomen and chest. A blood specimen was collected and plated on MacConkey agar, incubated at 37°C in ambient air. Lac- colonies grew on the plates. The cultured organism was a gram-negative rod that did not produce Shiga or Shiga-like toxins. The most likely etiological agent for this man's disease is: A. Enterohemorrhagic Escherichia coli B. Shigella dysenteriae C. Salmonella typhi D. Helicobacter pylori E. Campylobacter jejuni

*The answer is E.* The Haemophilus influenzae b (Hib) vaccine has had remarkable success in reducing the numbers of serious, life-threatening infections due to H. infl uenzae b. Infants are routinely vaccinated with Hib starting at 1 month of age. H. infl uenzae b is transmitted through respiratory droplets and causes a variety of diseases in people of various ages. Thus, avoidance of infants with meningitis would not have protected this infant from infection. Improved community sanitation (treatment and disposal of human waste, clean water, etc.) is not an issue in the transmission of this pathogen. The use of 10% bleach solution to clean toys shared with other infants is recommended to reduce infections and avoidance of day care facilities or other crowded situations could decrease exposure of the infant to infection with this agent. Neither of these options can replace vaccination as a way to prevent infection with H. infl uenzae b.

An 18-month-old male presents with fever, lethargy, malaise, productive coughing, and vomiting. Culture of cerebral spinal fluid samples with Staphylococcus aureus on blood agar reveals the presence of "satellite" colonies. Serology has indicated that the organism is polyribitol phosphate (PRP) positive. Gram stain of the colonies indicates the presence of a Gram-negative coccobacillus. How could the infection in the above case have been avoided? (A) Improved community sanitation (B) Avoidance of infants with similar symptoms (C) Avoidance of day care facilities or other crowded situations (D) Use of 10% bleach solution to clean toys shared with other infants (E) Vaccination of the infant

*The answer is B.* All the organisms listed could have caused pneumonia in this patient. M. pneumoniae causes a mild illness. Sputum production is scant with L. pneumophila. Rust-colored sputum is typical for S. pneumoniae. Thick, bloody sputum, described as "currant jelly" sputum, is characteristic of pneumonia caused by K. pneumoniae. This organism is an opportunistic pathogen causing lung infections in middle-aged and older individuals with alcoholism and patients with diabetes or chronic bronchopulmonary disease. Growth of the organism in the lungs causes necrosis, inflammation, abscess formation, cavitation, empyema, pleural adhesions, and hemorrhage leading to prominent bloody sputum. It is a severe illness with rapid onset of high fever and chills, flu-like symptoms, and a cough productive of abundantly thick, tenacious, blood-tinged sputum. Mortality rate is over 50% despite early and appropriate antimicrobial treatment. Risk factors for infection include a compromised respiratory immune system, hospitalization, intubation, diabetes, and alcoholism.

An 85-year-old male nursing home patient with a history of alcoholism suddenly developed a flu-like illness. He complained of chills and fever and had frequent coughing spells productive of thick, bloody sputum. The attending physician diagnosed bronchopneumonia and prescribed antibiotics, but regrettably the patient died within a week. What is the most likely cause of the patient's pneumonia? (A) Haemophilus influenzae (B) Klebsiella pneumoniae (C) Legionella pneumophila (D) Mycoplasma pneumoniae (E) Streptococcus pneumoniae

*The answer is C.* The clinical picture described fits the description of enteroinvasive E. coli that causes a Shigella-like dysentery. ETEC is a cause of traveler's diarrhea. EPEC is a major cause of childhood diarrhea in developing countries. EIEC causes Shigella-like dysentery. EHEC causes hemorrhagic colitis; EAEC (30% of the traveler's diarrhea cases) is primarily associated with persistent diarrhea in children in developing countries.

An adult tourist visiting a remote area of Guatemala developed fever, prostration, malaise, dysentery, and dehydration. Based on current epidemiologic data of the region, a local physician reported that the causative agent was probably a diarrheogenic strain of Escherichia coli and did not prescribe antibiotic therapy. Presuming the Guatemalan doctor to be correct, which type of virulent E. coli is most consistent with this clinical picture? (A) Enteroaggregative E. coli (EAEC) (B) Enterohemorrhagic E. coli (EHEC) (C) Enteroinvasive E. coli (EIEC) (D) Enteropathogenic E. coli (EPEC) (E) Enterotoxigenic E. coli (ETEC)

*The answer is A.* The combination of necrotizing pneumonia and an alcoholic patient suggests Klebsiella pneumoniae, and the laboratory data (Lac+ and a luxuriant capsule) are consistent. Although Serratia can cause pneumonia in compromised patients, necrosis is not a characteristic feature. Moreover, the organism ferments lactose slowly, if at all, and does not have a luxuriant capsule. Yersinia pseudotuberculosis is Lac- and rarely causes pneumonia. Pseudomonas aeruginosa can cause pneumonia in compromised patients but does not ferment lactose. Campylobacter fetus typically causes bacteremia and disseminated infections.

An older, alcoholic male develops severe, necrotizing lobar pneumonia. The organism is Lac+ and produces a luxuriant capsule. The most likely agent is: A. Klebsiella pneumoniae. B. Serratia species. C. Yersinia pseudotuberculosis. D. Pseudomonas aeruginosa. E. Campylobacter fetus.

*The answer is D.* Plague is a zoonotic infection of rodents caused by Yersinia pestis. It is transmitted among rodents and rarely to people by fleas. Human infection resulting from the bite of an infected flea results in bubonic plague characterized by high fever and painful, enlarged lymph nodes or bubos. Bubonic plague is not transmitted person-to-person. Bacteremia can occur in untreated individuals with bubonic plague, leading to lung infection or pneumonic plague. This form of plague is transmissible person-to-person through respiratory aerosols.

An outbreak of pneumonic plague occurs in the Democratic Republic of the Congo killing over half of the affected individuals. How is this form of plague transmitted? (A) Bite of infected flea (B) Direct skin contact with bubos (C) Ingestion of contaminated water (D) Inhalation of aerosolized bacteria (E) Introduction of bacteria into wounds while butchering infected animals

*The answer is D.* This is one of the most commonly used solid media, enabling differentiation of lactose-fermenting and nonfermenting enteric Gram-negative bacteria and inhibiting the growth of Gram-positive organisms and nonenteric Gram-negative organisms such as Neisseria. Blood agar allows for the nonselective growth of a broad range of nonfastidious bacteria. Buffered charcoal yeast extract agar is used to isolate Legionella. Chocolate agar is made with lysed sheep red blood cells and is used to isolate fastidious organisms like Neisseria and Haemophilus. Thayer-Martin agar is used to isolate Neisseria gonorrhoeae in specimens collected from nonsterile sites. This agar allows for the growth of N. gonorrhoeae while suppressing the growth of other bacterial contaminants in the specimens.

Blood and urine cultures are taken from a 73-year-old man hospitalized with suspected Escherichia coli urosepsis. Which culture medium would allow selective growth of Gram-negative enterics while suppressing the growth of Gram-positive bacteria, Haemophilus and Neisseria? (A) Blood agar (B) Buffered charcoal-yeast extract agar (C) Chocolate agar (D) MacConkey agar (E) Thayer-Martin agar

*The answer is A.* Uropathogenic E. coli frequently colonizes the bowel and contamination of the female urethra is common. Sexual activity can promote ascension of uropathogenic E. coli to the bladder to cause a UTI. Circumcision reduces the risk of E. coli UTIs in boys.

During a gynecologic office visit, a positive leukocyte esterase test of the urine along with a voiding symptom prompts the physician to order a urine culture. The bacterium, which was isolated in clinically significant numbers, grows profusely on MacConkey agar, as shown in the accompanying photograph, and ferments lactose. With reference to the above case, what is the most likely source and mode of transmission for this urinary bacterial isolate? (A) Contamination with perianal and bowel flora (B) Hematogenous spread from the kidneys (C) Inadvertent inoculation with commensal organisms on skin (D) Transmission from an uncircumcised sexual partner (E) Waterborne spread

*The answer is C.* At this stage, the diagnosis is dependent primarily on the clinical picture of the patient. Erythema migrans, often a bull's-eye appearance, is highly suggestive of Lyme disease. Technically, however, it is not considered pathognomonic as similar lesions occur with insect bites and other tickborne agents. Borrelia burgdorferi cannot be readily visualized from lesions' samples. Culture is impractical due to the fastidious growth requirements of the bacterium. Liver function tests are not helpful in making a diagnosis of Lyme disease. The probability of a positive serology is low, as seroconversion usually takes place several weeks following infection. When antibodies are present, diagnostic serology is based on a positive ELISA confirmed with a Western blot.

During the summer in western Massachusetts, a teenager complains of aching muscles and joints and mild fever. He notices a rash on the side of his upper leg (shown in the photograph), which prompts his mother to take him to the clinic. How would the physician determine the diagnosis in the above case? (A) CBC assessment and visualization of the microbe from the lesion by dark field microscopy (B) History and clinical presentation of the patient plus elevated liver function tests (C) History and clinical presentation of the patient alone (D) Culture of the organism from blood and synovial fluid (E) Serologic tests to detect antibodies in the patient serum against the organism

*The answer is C.* Most of the time, early Lyme disease responds well to the tetracyclines. The above patient is a teenager, with no pregnancy issues to contend with, and doxycycline is the appropriate choice. Misdiagnosed cases and failure to treat early in the course of the disease process could result in debilitating neurologic (e.g., meningitis, facial palsy) and cardiac complications (e.g., third-degree heart blocks, myocarditis, or pericarditis).

During the summer in western Massachusetts, a teenager complains of aching muscles and joints and mild fever. He notices a rash on the side of his upper leg (shown in the photograph), which prompts his mother to take him to the clinic. With regard to the above case, what is the drug of choice? (A) Ceftriaxone (B) Chloramphenicol (C) Doxycycline (D) Metronidazole (E) Streptomycin

*The answer is C.* Most O157:H7 Shiga-like toxin producing E. coli strains do not ferment sorbitol during overnight incubation in contrast to the approximately 80% of other E. coli strains that ferment sorbitol rapidly. Sorbitol-containing MacConkey agar is used for isolation of this strain. Additionally E. coli O157:H7 does not produce glucuronidase. Glucuronidase production is a characteristic of other strains of E coil. EHEC is recognized as a cause of hemorrhagic colitis most characteristically associated with ingestion of inadequately cooked hamburger meat. EHEC strains produce a toxin that is virtually identical to the Shiga toxin produced by Shigella dysenteriae. This Shiga-like toxin is phage encoded, and its production is enhanced by iron deficiency. The Shiga and the Shiga-like toxins inactivate the 60S ribosomal subunit in human cells leading to an inhibition of protein synthesis and eventual cell death. Infection with EHEC can also lead to the hemolytic-uremic syndrome (HUS) characterized by thrombocytopenia, microangiopathic hemolytic anemia and renal insufficiency (uremia). EHEC do not invade the intestinal mucosa: this is a characteristic of Enteroinvasive E. coil as well as other causes of hemorrhagic diarrhea. The organism does not produce LT or ST; these toxins are produced by Enterotoxigenic E. coil (ETEC).

E. coli isolated from a patient with diarrhea does not ferment sorbitol on sorbitol containing MacConkey agar and does not produce glucuronidase. The toxin produced by these bacteria would most likely: A. Activate adenylate cyclase B. Activate guanylate cyclase C. Inactivate ribosomal subunits D. Inactivate EF-2 E. Disrupt the cellular cytoskeleton

*The answer is D.* After ingestion, the bacterium adheres via fimbriae to M cells, a type of intestinal epithelial cell. The organisms then insert, via a type III secretion system, proteins that induce actin reorganization leading to M cell membrane riffling and engulfment of Salmonella. The organisms replicate with phagosomes, leading to M cell death and allowing further spread of the pathogen to neighboring epithelial cells and macrophages. Hence, Salmonella is correctly referred to as a facultative intracellular pathogen able to elude specific immune defenses and disseminate deeper into the organ or body. Other bacterial microbes that subjugate the macrophages include Legionella, Yersinia, Francisella, and Mycobacterium.

Fimbriae-mediated adherence to intestinal epithelial cells, type III secretion system-mediated endocytosis and transport to the lamina propria to elicit a localized inflammatory response are descriptive of the infectivity and pathogenesis of which diarrheogenic agent? (A) Campylobacter jejuni (B) Clostridium difficile (C) Enterotoxigenic Escherichia coli (D) Salmonella sp. (E) Vibrio cholerae

*The answer is D.* The case describes an outbreak of cholera characterized by abundant "rice water" stools and rapidly leading to dehydration. Control of the outbreak is essential and, where possible, definitive diagnosis of the etiologic agent is recommended and the organism is cultured on TCBS media. The other organisms listed typically cause bloody diarrhea.

Five adults and seven children in a village in South Africa present over several days to a local clinic with voluminous watery diarrhea containing flecks of mucus. They show signs of severe dehydration and are treated with oral fl uids and electrolytes. Stool cultures grew yellow colonies on thiosulfate citrate bile salt (TCBS) media. What is the etiology of this outbreak? (A) Enteroinvasive Escherichia coli (B) Salmonella typhi (C) Shigella dysenteriae (D) Vibrio cholerae (E) Yersinia enterocolitica

*The answer is D.* Based on the fever, headache, a petechial rash, myalgia, GI and pulmonary issues, and a history of tick exposure in a prime endemic area in the United States, instigation of antibiotic therapy for Rocky Mountain spotted fever (RMSF) is probably warranted. In the United States, RMSF is the most commonly diagnosed rickettsial illness. Rickettsia rickettsii, the causative agent of RMSF, is an obligate intracellular microbe that exhibits tropism for endothelial cells of the small blood vessels. An intracellular parasitic niche seems to be advantageous for a bacterium because it can afford some degree of protection against the killing activities of white blood cells. Nevertheless, the endothelial cells are eventually damaged and vasculitides will develop in many sites in the body. For example, the characteristic cutaneous rash is directly linked to vasculitis. Even though this case reads like a textbook, a clinical diagnosis is often difficult to make with a high degree of certainty because it may be confused with other diseases like meningococcal meningitis, secondary syphilis, measles, etc. The rash, which first appears on the wrists and ankles within days of being infected, eventually spreads centripetally. The rash is often difficult to detect in darker pigmented patients. Furthermore, the disease is potentially more severe in African Americans, possibly because of a higher genetic likelihood of a glucose-6-phosphate dehydrogenase deficiency. The possibility for adverse complications, including a fatal outcome and the fact that seroconversion may not occur until well into the second week, justifies initiating antibiotic therapy (e.g., doxycycline or chloramphenicol) quickly. Delaying it would put this patient at risk for a more severe course.

In early July, a middle-aged African-American male from rural southwestern Missouri is brought to regional urgent care facility. He has been vomiting most of the night and is currently febrile. Furthermore, he complains of "achy muscles" and a throbbing headache. Rales are detected on auscultation. Physical examination reveals a petechial rash on the forearms, and an engorged tick is removed from the groin area. Considering the particulars of this case, which of the following is the most crucial action that should be taken for successful management and a positive outcome? (A) Aggressive management of fl uid intake to prevent rehydration (B) Chest radiographic study to rule out interstitial pneumonia (C) Gram stain clean-catch urine sample for evidence of a urinary tract infection (UTI) (D) Immediate initiation of doxycycline (E) Submit a blood sample for immunodiagnosis to the lab

*The answer is B.* Infection with this strain of E. coli is associated with hemolytic uremic syndrome. This complication may be more likely to occur if the patient is treated with antibiotics. Cholera, typhoid fever, pseudomembraneous colitis, and shigellosis respond favorably to antibiotic treatment.

Intestinal infection with which of the following organisms should not be treated with antibiotics? (A) Clostridium difficile (B) Escherichia coli O157:H7 (C) Salmonella typhi (D) Shigella sonnei (E) Vibrio cholera

*The answer is A.* Of the diseases listed, bacillary dysentery is the only one that fits the description. This disease is caused by Shigella dysenteriae, a nonmotile, Gram-negative rod that does not ferment lactose. Shigella species causing human disease include S. dysenteriae, S. fl exneri, S. boydii, and S. sonnei. S. dysenteriae causes dysentery, a condition characterized by the passage of numerous small bloody stools containing mucus and pus. Humans are the only natural reservoir for Shigella. The organism is spread by ingestion of fecal contaminated food or water. The infective dose is extremely low; as few as 200 bacteria can initiate the infection. Shigellosis occurs worldwide. S. dysenteriae, S. flexneri, and S. boydii are found particularly in Asia, Africa, and Central and South America. They tend to occur whenever war, natural calamities (earthquakes, floods, etc.), or unhygienic living conditions result in overcrowding and poor sanitation. Shigella sonnei is the species found in developed countries such as the United States. It tends to cause watery diarrhea rather than bloody diarrhea.

Medical outreach physicians, working in a refugee camp in central Africa, are concerned with the number of young children presenting with high fever, bloody diarrhea, and dehydration. The crowded living conditions of the camp suggest person-to-person contact, but because of the remoteness of the site only limited laboratory studies are available. The suspected culprit is described as a nonmotile, Gram-negative bacillus that does not produce lactose-positive colonies on selective media. Based on these findings, which disease is more likely? (A) Bacillary dysentery (B) Campylobacteriosis (C) Cholera (O:1 classic biotype) (D) Nontyphoidal salmonellosis (E) Staphylococcal food poisoning

*The answer is A.* The case is descriptive of tickborne relapsing fever endemic to the western United States. Several spirochete species of the genus Borrelia are linked to this disease. Borrelia recurrentis, which causes another type of relapsing fever, is transmitted to humans by the Pediculus body louse and is found in certain areas of Africa and South America. Tickborne relapsing fever is usually successfully treated with antibiotics. Of the other organisms listed, only T. pallidum, the causative agent of syphilis, is a spirochete.

Over the course of a summer, several patients present to hospitals in the northwestern United States with complaints of fever, chills, headache, and other viral-like symptoms. Reportedly, with many of these patients, the fever subsides only to return with a vengeance later. Of significance is that spirochetes are observed in peripheral blood smears. Another common denominator is that all patients are outdoor enthusiasts and frequently engage in activities like camping, fishing, and hiking in mountainous areas. What is the most likely etiologic agent? (A) Borrelia hermsii (B) Treponema pallidum (C) Rickettsia rickettsiae (D) Proteus vulgaris (E) Vibrio vulnificus

*The answer is C.* Cattle are the reservoirs of Escherichia coli O157:H7; they show no signs of disease and shed the organism sporadically. Humans become infected when ingesting the organism in undercooked hamburger or foods contaminated with cattle fecal material. Children can also contract the organism by contaminating their hands through petting a cow. The organism is rarely isolated from chlorinated drinking water. E. coli O157:H7 can also be transmitted person-to-person in crowed settings such as day care centers; however, direct or indirect contact with cattle feces is a more likely route.

Physicians and state epidemiologists investigate a cluster of acute diarrheal cases in children who attend a preschool in a small Midwestern town. The children presented with fever, acute episodes of bloody diarrhea, and petechial rash or purpura. Two of the children have scanty urine output and are showing signs of renal failure. Fecal leukocytes and parasites are absent in microscopic smears, and no obvious pathogens are identified using standard microbiologic media. The preschool in the above case only serves pasteurized milk and packaged cookies to the children and epidemiologists quickly rule out a food source as the risk factor. With this in mind, which of the following is the most plausible means by which the children were infected? (A) Contagious transmission from an infected child (B) Drinking water of the preschool is contaminated with the microbe (C) Failure to wash hands after visiting a petting zoo (D) Fecal-oral transmission, most likely from the restroom (E) Visiting the elderly in an assisted-living or nursing home facility

*The answer is B.* This organism is associated with bloody diarrhea and hemolytic uremic syndrome (HUS), which is affecting two of the children. HUS is primarily a disease of infancy and early childhood caused by the production of a Shiga-like toxin that binds to and kills renal endothelial cells. HUS presents as a triad of microangiopathic hemolytic anemia and thrombocytopenia and can lead to acute renal failure. Diarrhea caused by E. coli O157:H7 is a precipitating factor and the use of antimotility drugs may increase the risk. The clinical picture of HUS can suggest a GI bleed because the stool may be grossly bloody. Urine output is reduced or absent with renal failure; petechiae, purpura, and fever are common findings. Schistocytes are markers of HUS. No E. coli was isolated from the blood because normal stool culturing does not reveal the organism (sorbitol-MacConkey medium is a screening that can give a preliminary identification). Antibiotics are not effective for EHEC. With the exception of B. cereus, which causes vomiting or watery diarrhea, all of the organisms listed can cause bloody diarrhea but none is associated with HUS. Shigella dysenteriae (not S. fl exneri) can also cause HUS through the production of Shiga toxin.

Physicians and state epidemiologists investigate a cluster of acute diarrheal cases in children who attend a preschool in a small Midwestern town. The children presented with fever, acute episodes of bloody diarrhea, and petechial rash or purpura. Two of the children have scanty urine output and are showing signs of renal failure. Fecal leukocytes and parasites are absent in microscopic smears, and no obvious pathogens are identified using standard microbiologic media. What is the most common infectious agent consistent with this clinical picture? (A) Bacillus cereus (B) Enterohemorrhagic Escherichia coli (EHEC) (C) Salmonella typhi (D) Shigella flexneri (E) Yersinia enterocolitica

*The answer is D.* The Haemophilus influenzae type b (Hib) vaccine consists of PRP (poly ribose-ribitolphosphate) derived from the capsule of H. influenzae type b coupled with either diphtheria or tetanus toxoid. This combination vaccine is now approved for use in infants with the initial series of vaccinations at 2, 4, and 6 months of age and with a booster dose atl5 months of age. Infants possess protective maternally derived transplacentally acquired lgG against Hib, but this protection fades as these immunoglobulins are degraded during the first few months of life. Infants and young children are unable to mount an adequate immune response against T-lymphocyte-independent antigens such as bacterial capsular polysaccharide. When the polysaccharide antigen is conjugated with diphtheria or tetanus protein toxoid the conjugate becomes a T-cell-dependent antigen, and the immunogenicity of the vaccine is thereby increased (Choice D). Infants and young children have been shown to be quite capable of mounting protective immune responses to T cell dependent antigens. The additional protection afforded by this protein-polysaccharide conjugate vaccine results from T-cell dependent stimulation of B-lymphocytes to undergo class-switching from 1gM to lgG and the production of memory B-lymphocytes that stand ready to fight Hib infection should it occur. Because of the conjugated vaccine there has been a tremendous decline in the incidence of invasive disease caused by H. influenzae type b.

The vaccine that provides protection against H. influenzae contains capsular polysaccharide of H. influenzae type b as well as diphtheria toxoid. Diphtheria toxoid in this vaccine: A. Improves safety B. Increases solubility C. Mounts diphtheria protection D. Increases immunogenicity E. Cross-reacts with human antigens

*The answer is B.* Very few cases of invasive diseases attributable to H. influenzae b occur in developed countries with high rates of vaccination. The incidence of invasive diseases due to H. influenzae b has decreased in the United States by 99% since the vaccine first became available in 1989. Although non-b serotypes of H. influenzae still cause disease, these strains are generally not involved in invasive diseases. Vaccines are also available for S. pneumoniae and N. meningitidis, and although the efficacy of these vaccines is still quite high, not all serotypes involved in disease are included in these two vaccines.

Vaccination against which bacterial species has been most successful at reducing the number of cases of invasive diseases, including meningitis? (A) Escherichia coli K1 (B) Haemophilus influenzae b (C) Listeria monocytogenes (D) Neisseria meningitidis (E) Streptococcus pneumoniae

*The answer is C.* Yersinia pestis can be transmitted to humans via the bite of an infected flea. The organism can also be transmitted person to person via the respiratory route if the lungs of the source patient or animal are infected. Brucella abortus infection is acquired via skin abrasions or ingestion. Pseudomonas aeruginosa is transmitted via direct inoculation into the respiratory tract, urinary tract, or into wounds in the hospital setting. Legionella pneumophila is acquired by inhalation of environmental ameba containing the bacteria. Free-living L. pneumophila, after multiplying in protozoa within water systems, can also be inhaled, resulting in Legionnaire disease or Pontiac fever. Pasteurella multocida is most commonly transmitted to humans via the bite of an infected animal.

Which of the following is transmitted to humans via an arthropod vector? A. Pseudomonas aeruginosa B. Legionella pneumophila C. Yersinia pestis D. Brucella abortus E. Pasteurella multocida

*The answer is A.* The capsule is antiphagocytic, and facilitates hematogenous dissemination of Haemophilus influenzae. Although H. influenzae is an important pathogen of infants and young children, passive transfer of maternal immunoglobulin G may afford neonates protection. Immunity begins to wane in older adults, increasing the risk of infection for this population. Humans are the only natural host for H. influenzae. H. influenzae requires both hemin, X factor, and nicotinamide adenine dinucleotide (NAD), V factor, which are not available in sheep blood agar. Heating the blood lyses the erythrocytes, releasing both X and V factors, and simultaneously inactivating an NAD-inactivating enzyme present in blood. Media made with such heated blood is termed "chocolate agar." The organism does prefer elevated CO2.

Which of the following is true of Haemophilus influenzae? A. Invasive infections are most commonly associated with encapsulated strains. B. Most invasive infections occur in infants during the neonatal period. C. Most human infections are acquired from domestic pets. D. The organism can be readily cultured on sheep blood agar in an environment of elevated CO2. E. Older adults are rarely at risk for infection with this organism because they typically have a high level of immunity.

*The answer is B.* Bordetella pertussis typically causes a lymphocytic leukocytosis. Initial symptoms of Bordetella infection are relatively nonspecific (rhinorrhea, etc.). The characteristic paroxysmal coughing begins somewhat later. Maintenance of solid immunity depends on repeated exposure to the organism, either through natural causes or by administration of booster shots. Growth of Bordetella requires a medium containing a substance such as charcoal to absorb or neutralize inhibitory substances and also antibiotics that inhibit the growth of normal flora.

Which of the following statements about Bordetella pertussis infection is true? A. Infection causes a leukocytosis characterized primarily by a marked elevation in polymorphonuclear leukocytes. B. Isolation of the organism from clinical specimens is greatest during the early stages of illness. C. Clinical diagnosis of whooping cough can usually be made within a few days of onset of initial symptoms. D. Children who receive a full series of immunizations with the pertussis vaccine generally develop solid, lifelong immunity to pertussis. E. The organism can be cultured on standard laboratory media such as sheep blood agar.


Set pelajaran terkait

Units of Production Depreciation Method

View Set

Language Arts: Quiz 3: Analyzing Story Structure; Comparing and Contrasting Stories

View Set

الكيمياء للصف الثالث الثانوي - علل

View Set

INNOVATION&CHANGE: Chapter 7 Quiz

View Set